Сохранен 507
https://2ch.hk/sci/res/564437.html
24 декабря Архивач восстановлен после серьёзной аварии. К сожалению, значительная часть сохранённых изображений и видео была потеряна. Подробности случившегося. Мы призываем всех неравнодушных помочь нам с восстановлением утраченного контента!

Тред тупых вопросов

 Аноним 01/09/22 Чтв 07:31:21 #1 №564437 
doc2020-04-1621-28-12.mp4
Даем умные ответы на тупые вопросы
Аноним 01/09/22 Чтв 07:31:40 #2 №564438 
>>564437 (OP)
что происходит на гифке
Аноним 01/09/22 Чтв 07:40:07 #3 №564439 
>>564438
мамку твою ебут
Аноним 01/09/22 Чтв 07:48:09 #4 №564440 
сентябрь.mp4
>>564439
сын шлюхи, тебя парта ждет, беги уже
Аноним 01/09/22 Чтв 08:49:26 #5 №564445 
>>564438
>что происходит на гифке
Белок-кинезин тащит везикулу по микротрубочке.
Аноним 01/09/22 Чтв 09:06:39 #6 №564446 
>>564440
не рвись, учоный
Аноним 01/09/22 Чтв 09:35:58 #7 №564447 
>>564445
Куда тащит и зачем? Почему ему никто не мешает
Аноним 01/09/22 Чтв 10:22:37 #8 №564452 
>>564447
>Куда тащит и зачем? Почему ему никто не мешает
Куда надо, туда и тащит. В везикуле какие-то вещества, необходимые для каких-то клеточных процессов. На тот случай, если нужно, чтобы они оказывались в нужном месте быстрее, чем своим ходом за счет броуновского движения, в клетке есть система митротрубочек и белки, способные по ним передвигаться.
Аноним 01/09/22 Чтв 10:25:34 #9 №564453 
>>564446
мой пиструн рвет твой девственный анус
Аноним 01/09/22 Чтв 11:31:39 #10 №564460 
EBinAisWkAAPwf.jpg
>>564453
Аноним 02/09/22 Птн 07:23:14 #11 №564510 
Интересует общая причина, почему охлаждать в целом куда труднее и дольше, чем нагревать. Например, нагреть поверхность венеры ещё на 500 градусов проще простого - ёбнуть астероидом. Охладить за то же время (пару часов), конечно, тоже можно, но либо медленно спустив туда лёд, либо разобрав планету на фрагменты, охладить их по отдельности и вернуть обратно,- незоизмеримо большие энергозатраты. Либо оградить от солнца и ждать 100 лет. Каковы общие физические соображения по поводу этой асимметрии? Например, для нагрева достаточно точечного воздействия, для заморозки - нужно воздействие на всю поверхность и работы в окружающем пространстве.
Аноним 02/09/22 Птн 07:58:41 #12 №564511 
>>564510
Верхний предел энергии и температур не ограничен. Нижний ограничен абсолютным нулем. Можно нагревать тело температурой миллион градусов, но нельзя охлаждать минус миллионом.
Аноним 02/09/22 Птн 09:04:25 #13 №564517 
Вот тем, кому 50+, не стыдно работать в университете, не имея докторской? Схуяль они на кандидатской-то останавливаются? Чтобы в 70 лет доцентом ходить как ЛОХ?
Аноним 02/09/22 Птн 11:49:43 #14 №564524 
>>564510
Скорость теплопередачи(а это в той или иной степени основной метод охлаждения) падает с понижение температуры.
Можно переформировать через энтропию. По определению dS=dQ/T dQ тут не дифференциал, что не придрались всякие моченые. Как правило энтропия растет с температурой, и следовательно чтоб охладить тело нужно уменьшить его энтропию. И тут мы замечаем, что при изменение энтропии при низких температуры нужно передать теплоты больше чем при высоких. Вот так и получается, что охлаждение идет труднее.
Аноним 02/09/22 Птн 12:08:42 #15 №564525 
>>564524
А вот интересно, чем тяжелее частица, тем она более теплоёмкая.
Но температура, передача импульса, происходит как столкновение
Но сталкиваясь с тяжёлой медленной частицей, её скорость почти не изменяется, как и почти не изменяется скорость столкнувшейся с ней горячей быстрой частицей.
Получается теоретческое идеальное вещество бесконечно большой теплоёмкости имело бы бесконечно малую теплопроводность.
Аноним 02/09/22 Птн 13:07:42 #16 №564526 
>>564525
>чем тяжелее частица, тем она более теплоёмкая.
Нет.
В общем случае теплоемкость зависит только от количества включенных степеней свободы, а не от массы. В случае газа, теплоемкость зависит только от количество частиц. И один моля гелия и один моль радона обладают примерно одинаковой теплоемкостью, хотя их массы различаются в 55 раз!
Так и выходит, что чем легче газ, тем больше у него теплоемкость. У водорода самая большая теплоемкость на пересчет массы. У нейтрино должны быть еще больше.
Теплопроводность в общем случае зависит от диффузии, а она уже зависит от того как разные степени степени свободы включатся или выключается. В случае газа, теплопроводность зависит от включаемых степеней свободы соответствующих движению в пространстве. Ну и соответственно зависит только от молей и температуры.
Аноним 02/09/22 Птн 14:59:54 #17 №564531 
0ii76b1w-4g4pPN8pRt1vgGURL9m9fhTMODEh0z5bN9uBAtuGlnX3ZBPHo0AYpqt9tCy3zyqx5ZgA-anPfJDZx6dx6ZBqmynTYr0GbLDWv6Ulnzp-6yCWEnU9dhBv.jpg
>>564525
Аноним 02/09/22 Птн 16:35:23 #18 №564533 
>>564437 (OP)
https://www.techcult.ru/11024-rasteniya-v-temnote
Что именно они здесь называют ацетатом? Хочу эту новость проверить дома на практике с рассадой помидоров.
Аноним 02/09/22 Птн 16:39:14 #19 №564534 
>>564533
Ацетат это ион уксусной кислоты CH3COO−
Аноним 02/09/22 Птн 17:17:06 #20 №564536 
>>564524
>dS=dQ/T
>И тут мы замечаем, что при изменение энтропии при низких температуры нужно передать теплоты больше чем при высоких
Я нихуя не понял что ты там заметил. Например если dQ=100 и T=10, то
dS=100/10=10
а если dQ=1000 и T=100, то получается
dS=1000/100=10
то есть для изменения энтропии на 10 при Т=10 нужно в 10 раз меньше энергии, чем при Т=100. Или ты что-то другое имел в виду? Просто у тебя все слова в разных падежах
>при изменение энтропии при низких температуры
Аноним 02/09/22 Птн 21:06:02 #21 №564546 
>>564534
И как они его получают электролизом из воды и углекислого газа? Растению на такой подкормке слишком кисло не будет?
Аноним 02/09/22 Птн 21:57:25 #22 №564549 
>>564536
Я действительно хуево объяснил. Попробую по другому.
Теплота не есть энергия. Теплота это показывает сколько энергии было "выровнено" между системами при теплообмене.
Энтропия по определению показывает эффективность этого "выравнивания". И из соотношения в другом виде показывает dQ=TdS показывает эффективность теплообмена.
Короче dQ это сколько теплоты будет передано между системами при акте теплообмена при данной температуре T. Чем ниже температура, тем меньше будет передано теплоты между системами.
Аноним 03/09/22 Суб 00:30:27 #23 №564555 
А как дерево или растение может эволюционировать в животное или насекомое?
Аноним 03/09/22 Суб 00:33:37 #24 №564556 
>>564555
А они могут, разделение же еще на этапе колоний бактерий произошло? Хотя, хищные растения есть, животные с хлорофиллом тоже, так что кто знает.
Аноним 03/09/22 Суб 00:37:47 #25 №564557 
>>564556
Я хз какой там последний общий предок.

Но вот например водоросли стали чем-то типа вируса или паразита.
Хз ближе вирус к животному чем растение или нет.
Аноним 03/09/22 Суб 09:26:01 #26 №564561 
Из абортосрача вопрос: так ли однозначно можно определить момент смерти раннего эмбриона? Можно ли искусственно поддерживать активность его клеток, чтоб формально считать его живым, чтоб не считать такой аборт убийством? Если поддерживать жизнь нескольких клеток, а остальные померли (или вообще вырезать их), плод ещё будет живым (хотя это уже говнософия пошла)?
Аноним 03/09/22 Суб 12:00:04 #27 №564564 
>>564561
>искусственно поддерживать активность его клеток, чтоб формально считать его живым, чтоб не считать такой аборт убийством
Тогда убийством будет тот момент, когда ты перестаешь поддерживать эту самую активность. А вообще, в чем смысл затеи? Весь смысл аборта в том, чтобы не было эмбриона. Зачем ты хочешь что-то там поддерживать?
Аноним 03/09/22 Суб 12:39:32 #28 №564565 
>>564564
> А вообще, в чем смысл затеи?
В том, что некоторые считают аборт убийством, по тяжести сравнимым с другими. Даже извлечение только что оплодотворённой яйцеклетки. Отметают все критерии о наличии сознания, чувстве боли и тд. Эмбрион - отдельная человеческая особь, неспособная к существованию вне матки, следовательно, аборты нужно радикально запретить (здесь не место этому срачу). Остаётся тогда вопрос, а что есть в принципе убийство человека. Если сохранить живой одну клетку, это не убийство получается, по логике антиабортников? А если изобрести средство, которое внутри матки будет убивать все клетки плода, кроме одной? Всё равно же всё упирается в возможность поддержания деятельности мозга как вместилища сознания/души, а не в сохранение уникального генотипа.
Аноним 03/09/22 Суб 12:50:44 #29 №564566 
>>564565
>Если сохранить живой одну клетку, это не убийство получается, по логике антиабортников?
Нет. Но право на жизнь - не единственное право, Анон.
Аноним 03/09/22 Суб 12:50:49 #30 №564567 
>>564565
Не трать время на этот тупорылейший срач, это проблема социологическая, а не биологическая или моральная или юридическая.
Аноним 03/09/22 Суб 12:52:32 #31 №564568 
>>564565
В том что ты написал уже противоречие.
>некоторые считают аборт убийством... Даже извлечение только что оплодотворённой яйцеклетки
>если изобрести средство, которое внутри матки будет убивать все клетки плода, кроме одной?
а с этой одной ты что будешь делать? Ведь даже извлечение одной яйцеклетки тоже "убийство".
И опять же, если кто-то считает удаление 1 клетки убийством, то как ты собираешься это исправить убийством всех клеток кроме одной?
Это шиза сельдей. Не лезь в эту помойку. Серьезно. Иначе потратишь жизнь на подсчет количества гендеров и обоснование за какую команду должны выступать трансы.
Аноним 03/09/22 Суб 13:02:25 #32 №564569 
>>564566
>>564568
Да мне не сами аборты интересны, просто я тут чую какую-то подмену понятий, разобраться хотет. Они применяют юридически-бытовое понятие убийства (основанное на идее души/сознания, как я думаю) к биологическим процессам, к убийству как устранению данного генотипа из природы. В рамках науки таки можно спросить: есть ли биологический критерий убийства человека, совпадающий с бытовым и применимый к эмбриону в любой стадии развития?
> а с этой одной ты что будешь делать?
Неважно, жизнь одной клетки наверно можно поддерживать, давать ей делиться, уничтожая одного из двух потомков (или вообще к спидозникам пришивать). А если сейчас нельзя, то в будущем можно, наверно тут нет принципиальных ограничений.
Аноним 03/09/22 Суб 13:08:25 #33 №564571 
>>564569
>чую какую-то подмену понятий
Тогда тебе надо с этим не в научный раздел пиздовать а в какое-то другое место.

А так конечно противоречие есть, пару клеток говорят это ребёнок уже, убивать нельзя, аборт нельзя, но если достать пару клеток так чтобы они живые были, то это уже не ребёнок, а убийство. Хотя что так что так они живые.
Но им ни что не мешает просто сказать доставать нельзя я так скозал это закон!
Так что иди нахуй с этой диалектикой в другое место.
Аноним 03/09/22 Суб 13:13:19 #34 №564572 
>>564571
> Но им ни что не мешает просто сказать доставать нельзя я так скозал это закон!
> Так что иди нахуй с этой диалектикой в другое место.
с чего ты взял, что я этого придерживаюсь и призываю отстаивать? я спрашиваю, есть ли другой, научный критерий, с этим вопросом я пришёл.
Аноним 03/09/22 Суб 13:13:55 #35 №564573 
>>564571
Убийством будет помещение человека в условия, где он не может выжить. Игры с живыми клетками будут уже другим преступлением.
Аноним 03/09/22 Суб 13:14:21 #36 №564574 
>>564569
>а с этой одной ты что будешь делать?
>Неважно
В смысле неважно? Ты убиваешь все клетки кроме одной сейчас, а эту одну завтра? Ну и кого ты этим собираешься наебать?

>А если сейчас нельзя, то в будущем можно
Вот когда это будущее наступит, тогда и будешь разбираться, с учётом той реальности в которой будет это будущее. Тебе уже сказали, что это вопрос не биологии, а социологии и того каким образом определяются термины человек, эмбрион, убийство, не говоря уже о душе/сознании.
Аноним 03/09/22 Суб 13:17:10 #37 №564575 
>>564572
>с этим вопросом
Тебе и сказано что вопрос не научный, поэтому пиздуй в параши всякие.
Аноним 03/09/22 Суб 13:39:07 #38 №564576 
>>564574
> Ты убиваешь все клетки кроме одной сейчас, а эту одну завтра?
ну вот завтра и будут судить за убийство. И это завтра можно отодвигать хоть на 500 лет особенно если ввести мутацию на устранение теломер или содержать стволовую клетку (ну или как там им обеспечить бесконечное число делений). Для антиабортников принципиально, чтоб доктора и мамку судили сразу по извлечении плода независимо от его состояния на данный момент.
> Тебе и сказано что вопрос не научный, поэтому пиздуй в параши всякие.
Существуют ли биологические предпосылки социальных феноменов - ненаучный вопрос? Да ладно? А вечный здешний тред биологического бессмертия что тут делает тогда?
Аноним 03/09/22 Суб 13:41:09 #39 №564577 
>>564576
Пиздобол, ты не об этом спрашивал.
Естественно существуют предпосылки, ведь всё социальное детерминировано биологическим.
Аноним 03/09/22 Суб 13:44:26 #40 №564578 
>>564577
> Пиздобол, ты не об этом спрашивал.
Как раз об этом:
> есть ли биологический критерий убийства человека, совпадающий с бытовым и применимый к эмбриону в любой стадии развития?
Потому что сейчас смерть рождённого определяют через процедуру, которую не применишь к одноклеточному эмбриону. А можно ли изобрести применимую к обоим (кроме полного уничтожения генотипа)?
Аноним 03/09/22 Суб 13:46:48 #41 №564579 
99.9999% учоных в говне мочоных не смогут без подсказки из интернета ответить, каким образом светлячки зажигаются синхронно не видя друг друга.
Аноним 03/09/22 Суб 13:52:54 #42 №564582 
>>564576
>ну вот завтра и будут судить за убийство
Погоди, погоди. Изначально было

>>564561
>Из абортосрача вопрос
>>564569
>жизнь одной клетки наверно можно поддерживать, а если сейчас нельзя, то в будущем можно

И тебе ответили, что когда наступит такое будущее, тогда и будешь разбираться, а сейчас в абортосрачах ты этим аргументом никого не убедишь, потому что нет разницы убиваешь ты все клетки сразу или по одной в течении недели. А к чему ты про суд вообще заговорил? Аборты, по крайней мере в России, не являются преступлением, о каком суде вообще речь?
Аноним 03/09/22 Суб 13:54:06 #43 №564583 
>>564578
Пиздобол, это не предпосылки.
Твоё место в параше.
Аноним 03/09/22 Суб 13:55:23 #44 №564584 
>>564578
>смерть рождённого
Так ты же про аборты говорил, а не о новорожденных. У вас там в ваших абортосрачах у всех такая каша в голове?
Аноним 03/09/22 Суб 14:03:03 #45 №564585 
>>564583
это предпосылки: есть социальный феномен считать аборт убийством, а есть нет. у чего из этого есть научное обоснование, совпадающее с научным обоснованием обычного убийства?
> Твоё место в параше.
так я уже здесь, вон с тобой говном перебрасываюсь
> Пиздобол
а ты мудак и хамло и б, что гораздо хуже.
>>564582
ок, любая успешная пересадка органа ведёт к сохранению чужого генотипа, так что этот вопрос решён уже сейчас.
>>564584
Нет, я про тех и про других относительно общего критерия убийства.
Аноним 03/09/22 Суб 14:19:21 #46 №564586 
>>564585
>у чего из этого есть научное обоснование, совпадающее с научным обоснованием обычного убийства?
Ни у чего, сказали уже. Просто бабы всю историю морили детей, душили новорожденных, пили разное говно, чтобы произошел выкидышь. Вот и все "научное обоснование". Это не наука, это реальность того мира в котором мы все живём. Где-то это криминализируют, дают сроки, где-то просто казнят селедок насмерть, а где-то используют делают аборты в клиниках, чтобы сельди не травили себя всяким говном. Что неясного? В абортосрачах ты никого ничем не убедишь, потому что нет четкого критерия даже для того, какой срок считать справедливым для обычного убийства, где-то за убийство положена смертная казнь, где-то лет 5 могут дать и через 3 досрочно освободить. А ты думаешь, что можно как-то обосновать права группы коеток, да ещё и на основе рассуждений о душе. Тебе действительно лучше перейти на какой-нибудь профильный аборторесурс. Уверен, вы там много подобных "аргументов" совместно найдете, а тут тебя только хуями дальше будут кормить.
Аноним 03/09/22 Суб 14:38:37 #47 №564587 
>>564585
> это предпосылки: есть социальный феномен считать аборт убийством, а есть нет. у чего из этого есть научное обоснование, совпадающее с научным обоснованием обычного убийства?

Убийство это юридическое понятие. Что является убийством, а что нет определяет государство законами. В РФ аборт не убийство вот и весь ответ. Душа, нервная система и научное обоснование тут вообще не причём. Жизнь человека принадлежит государству и смертная казнь это тоже не убийство.
Аноним 03/09/22 Суб 18:08:32 #48 №564590 
ДЕРЕАЛИЗАЦИЯ


НЕ МОГУ ПРИНЯТЬ МИРОЗДАНИЕ


я не гумантиарий, верун и проч обызвательств
я физмат с дествам
учусь на физфаке

и все ранво порой опднимаю голову и смотр на солнце и думаю... блять, это гигиантский тяжеленый шар(обхект) с супер-гравитацией и супер термоялренымиреацкциями, который крутит нашу планету вокруг себя и греет её.
что
блять
такое

ЭТо правда? как?
как проверить? как осознать? как дотронуться?
аж голова болит и хочется уснуть и проснуться без мыслей об этом, а погоней за хвостами(я троечник), телками(я листва) и фантазями(рдр2 уже 2..нет 3... года откладываю)
Аноним 03/09/22 Суб 20:12:00 #49 №564596 
>>564590
Тебе в /psy
Аноним 03/09/22 Суб 20:30:12 #50 №564597 
В короткозамкнутом сверхпроводнике, бесконечно малое переменное магнитное поле не вызывает бесконечный ток.
Почему?
Ток, это количество летящих электронов. Раз сопротивления нет, что мешает полететь бесконечно большому количеству электронов и бесконечно быстро, так чтобы поток по количеству электронов через сечение был бесконечно большим?
Конечно их там столько просто нету по количеству, электронов, количество ограничено их в материале, и скорость полёта бесконечной быть не может, но всё же...

Бля, стоп, а в сверхпроводнике электроны вообще движутся?
Аноним 03/09/22 Суб 21:26:34 #51 №564601 
Screenshot121.png
Во что должна эволюционировать птица, чтобы образовать новую группу и быть выписанной из птиц. Я так полагаю это должно быть связано с потерей способности летать. Что еще
Аноним 03/09/22 Суб 22:04:45 #52 №564603 
>>564601
Пингвин остается птицей.
Аноним 03/09/22 Суб 22:12:16 #53 №564604 
>>564603
Вопрос какие еще мутации нужны, чтобы можно было говорить, например, о новом отряде
Аноним 03/09/22 Суб 22:38:35 #54 №564608 
>>564604
А что говорит определение отряда в Википедии?
Аноним 03/09/22 Суб 22:48:46 #55 №564610 
>>564608
Если бы у тасманийцев стабильно рос 4 моляр, то эта была бы заявочка на отдельный род. Но они вымерли
Аноним 03/09/22 Суб 23:02:35 #56 №564612 
>>564610
Это достаточный или один из нужных критериев?
Мне кажется невозможно признавать другим отрядом, когда это один вид.
Аноним 03/09/22 Суб 23:07:19 #57 №564613 
>>564612
Просто у других людей не было 4 моляра. Кроме зубов там много чего еще
Аноним 03/09/22 Суб 23:41:44 #58 №564615 
>>564613
>Кроме зубов там много чего еще
Вот видишь, смысла нет говорить что они могли стать другим отрядом, если бы они оставались при этом одним видом.
Аноним 03/09/22 Суб 23:46:03 #59 №564616 
>>564615
Так я не шарю в теме, просто повторил слова Дробышевского. Вопрос про птиц в силе
Аноним 04/09/22 Вск 13:02:03 #60 №564641 
1645652440685.gif
Новые данные телескопа Джеймса Уэбба свидетельствуют о том, что Большого Взрыва не было

https://etm-club.site/novye-dannye-teleskopa-dzhejmsa-uebba-svidetelstvuyu/?fbclid=IwAR2Rl6WbY-GKRtq_yknGpsN4-zuJc1Kzg7HFglfGBSn__0KSXcP2HaBMPz0

Как это понимать?
Аноним 04/09/22 Вск 13:09:40 #61 №564642 
>>564641
Хуя ты помойку откопал, конечно.
Аноним 04/09/22 Вск 13:20:49 #62 №564645 
>>564642
ага https://2ch.hk/news/
Аноним 04/09/22 Вск 14:02:34 #63 №564652 
>>564616
Это не имеет смысла. Вид разный/не разный знаешь, остальное просто болтология, все эти отряд и прочее, кто как нафантазировал, поэтому разбираться в этом не имеет смысла, пустая херня.
Аноним 04/09/22 Вск 14:47:32 #64 №564654 
16621617719220.gif
>>564652
Я просто хочу имаджинировать во что превратится птица через миллион лет. Они же совсем не похожи на своих предков, кроме яйцеклада. Какие дальнейшие мутации возможны. Страусы руки отрастят
Аноним 04/09/22 Вск 15:09:09 #65 №564662 
>>564654
>имаджинировать во что превратится птица через миллион лет.
Зависит от условий, может такой же птицей останется.
Миллион лет это не много, особо прям совсем сильных изменений не будет. Могут стать не летающими птицами, а те что были не летающими могут стать летающими. Руки в виде лап когтистых с перьями, возможно, как у тиранозавра.
Ты почему-то думаешь, что обязательно раз время идёт, значит должны во что-то другое превращаться.
Аноним 04/09/22 Вск 15:31:16 #66 №564666 
image.png
image.png
image.png
>>564662
А что, климат перестал меняться? Для точки отсчета берем страуса и пингвина, как наиболее ушедшие от стандартов. Может страус начнет превращаться в динозавра, а пингвин в морской обитателя
Аноним 04/09/22 Вск 15:36:58 #67 №564668 
>>564666
Как будет изменяться климат, условия окружающей среда (хищники, кормовая база и т.п.) и какие мутации будут происходить невозможно предсказать.
Аноним 04/09/22 Вск 15:50:19 #68 №564670 
image.png
image.png
image.png
image.png
>>564668
Можно нафантизовать если шаришь в сабже. Но я не шарю, думал встретить ученых тут
Аноним 04/09/22 Вск 16:56:07 #69 №564680 
>>564670
>Можно нафантизовать
С этим говном в b/
Аноним 04/09/22 Вск 20:29:57 #70 №564686 
>>564680
ну и пиздуй в быдлятню быдло, а кто шарит, тот ответит
Аноним 04/09/22 Вск 20:43:17 #71 №564687 
>>564686
Нет, попиздуешь ты, потому что твой вопрос это ненаучное говно для б/
Аноним 05/09/22 Пнд 08:27:42 #72 №564700 
Пральна понимаю, что все атомы состоят из протонов, нейтронов и электронов, и вся материя состоит из двух-трёх элементарных частиц, а свойства материи просто зависят от количества этих частиц и структуры атома?
Аноним 05/09/22 Пнд 08:38:56 #73 №564702 
>>564700
Не правильно
Аноним 05/09/22 Пнд 08:43:41 #74 №564703 
ДИСТИЛЛЯТОР.jpg
рЕКТИФИКАТОР.jpg
>>564437 (OP)
обьясните разницу между дистилляцей и ректификацией

я никак не пойму вот вообще.

вот википедия.
Слева статья дистиллятор
Справа статья ректификатор

Пики одинаковаые. Куб, колонна охладитель, слив.

я правда не пому
Аноним 05/09/22 Пнд 09:13:09 #75 №564704 
>>564703
Ну а почему же ты дальше картинок не смог? Прочитал бы хоть определение.
>Ректификация - это процесс разделения двойных или многокомпонентных смесей за счёт противоточного массообмена между паром и жидкостью. Ректификация - разделение жидких смесей на практически чистые компоненты, различающиеся температурами кипения, путём многократного испарения жидкости и конденсации паров.

А теперь смотри на свои рисунки.
Дистилляция это однократный цикл испарения и конденсации.
В ректификационной колонне внизу постоянно горячая температура, а вверху холодная. Причем выбраны они так, чтобы нужное вещество испарялось внизу, а конденсировалось где-то в середине, ну или ближе к верху, как показано у тебя. Жидкость испаряется внизу, по мере подъема конденсируется, сконденсировавшийся туман снова начинает опускаться вниз, где-то по дороге снова испаряется и так по кругу. В итоге получается, что ректификация это многократная дистилляция + разделение по температурам кипения, т.к. на разной высоте одной и той же колонны конденсируются фракции, отличающиеся температурой кипения. И как результат гораздо более чистый продукт.
Аноним 05/09/22 Пнд 09:16:26 #76 №564705 
А стабильные изотопы прямо вообще никогда не распадутся (по кр мере до распада протона в них) или таки могут, но очень нескоро? Тот же гелий-4. Не в том ли критерий стабильности, что вероятность а/б/гамма распадов не выше, чем всякой экзотики типа вылета рандомных нуклонов, захвата электронов и тд?
Аноним 05/09/22 Пнд 09:45:04 #77 №564706 
2qed.jpg
>>564704
спасибо
Аноним 05/09/22 Пнд 10:59:35 #78 №564707 
>>564705
Все одиночные стабильные ядра без внешнего воздействия условно вечны.
Аноним 05/09/22 Пнд 13:35:31 #79 №564708 
>>564702
А как правильно тогда, в чём ошибка?
Аноним 05/09/22 Пнд 14:15:56 #80 №564711 
>>564708
Правильные сказать, что материя состоит из кварково-лептонных полей + к ним калибровочные поля.
Ошибка состоит в том, что рассмотрение ядра как месива нейтронов и протонов, скрепленные юкаваскими силами, наивная и упрощенная модель в которых не описывается целые классы явлений.
Аноним 05/09/22 Пнд 14:26:24 #81 №564713 
>>564711
И тем не менее атомы однородны, и состоят из одних и тех же мельчайших частиц, и отличаются атомы между собой только порядком, количеством и структурой всех этих сил и частиц?
Аноним 05/09/22 Пнд 15:21:48 #82 №564719 
>>564713
На малом уровне частиц нет, совсем никаких. Частицы это условности нашего восприятия, которые на работают на микроуровне.
Корректнее говорит о дискретных порциях различных полей, которые в совокупности образуют структуры, которые мы уже называем атомами.
Аноним 05/09/22 Пнд 15:24:21 #83 №564720 
image.png
image.png
>>564708
у тебя понятие о материи времен Ньютона
Аноним 05/09/22 Пнд 15:55:26 #84 №564723 
>>564720
>>564719
Пидздец ебанутые, нахуй вы атом всё дальше и дальше делите, кукаретики? В микроскопе же дальше атома не увидишь, так нахуя плодить сущности? Бритва Оккама, не?
Аноним 05/09/22 Пнд 16:17:35 #85 №564726 
world.gif
Если человечество вернуть в каменный век, но со всеми нынешними знаниями, сколько потребуется времени, чтобы в существующее состояние развития техники вернуться?
Аноним 05/09/22 Пнд 16:23:07 #86 №564727 
>>564723
срыгни с треда, быдло. все равно тебе всю жизнь курьером вкус-очка работать
Аноним 05/09/22 Пнд 16:24:09 #87 №564728 
>>564723
Тащемта поход основанный на полях и симметриях требует меньше сущностный, чем классическая физика с частицами и атомами.
Аноним 05/09/22 Пнд 16:24:10 #88 №564729 
>>564726
Сотни лет
Аноним 05/09/22 Пнд 17:07:20 #89 №564732 
image.png
image.png
>>564726
смотришь на южную корею конца 19 века и конца 20 века, период индустриализации там занял 70 лет, если вычеркнуть оккупацию.
Аноним 05/09/22 Пнд 17:11:01 #90 №564733 
>>564727
Кукаретик, спок.
Аноним 05/09/22 Пнд 17:19:45 #91 №564737 
>>564732
Вот только они не сами создавали из камней и палок, а пользовались охуительными вливаниями и импортом. С тем же успехом можно высадиться на полярную шапку и устроить там палаточную деревню
Аноним 05/09/22 Пнд 17:46:32 #92 №564739 
>>564723
>Бритва Оккама, не?
Какой же ты тупорылый, лол.
Тут два варианта, или это троллинг, или ты фелосаф.
Аноним 05/09/22 Пнд 17:53:45 #93 №564740 
>>564733
почему ты еще не свалил, надеешься поумнеешь? лишняя хромосома не лечится, так что пиздуй
Аноним 05/09/22 Пнд 18:42:49 #94 №564742 
>>564739
>Какой же ты тупорылый, лол.
Аргументируй.

>>564740
Кукаретик, если ты не хочешь чтобы твой манямирок порушился, то это тебе стоит съебать, а не мне.
Аноним 05/09/22 Пнд 19:04:19 #95 №564743 
>>564723
>дальше атома не увидишь
Чего ты не видишь, того не существует, что ли, фаллософ мамкин?
Аноним 05/09/22 Пнд 19:52:48 #96 №564744 
>>564743
Чего не вижу - то не факт что существует, и нужны существенные доказательства, понятные лично тебе и доступный эксперимент, чтобы решить, что такое существует. А ты задрочил теорию и как тупой верун веришь в то, что атом делим, и что делим он именно так, а не как то иначе.
Аноним 05/09/22 Пнд 22:55:37 #97 №564746 
>>564723
>дальше атома не увидишь
Через фотоны не увидишь. А через гравитацию?
Аноним 06/09/22 Втр 03:34:34 #98 №564749 
>>564744
Советую загуглить "метрия" и почитать хотя бы пару минут. А то ты настолько дебил, что даже примерно не понимаешь что тебе говорят
Аноним 06/09/22 Втр 09:43:14 #99 №564753 
image.png
Какие твёрдые научные доказательства, что ихтиозавры были рептилиями, а не амфибиями?
Аноним 06/09/22 Втр 15:46:49 #100 №564774 
image.png
Внимание, возможно платина.

Летят в коцмицком просранстве две коробли.
Расстояние меж ними 300.000 км.
Скорость у их обоих = скорость света.
Один другому подаёт сигнал (поворотником) мигая раз в пол секунды.
Второй увидит сигнал?
Аноним 06/09/22 Втр 19:19:34 #101 №564783 
>>564774
Просто представь камешки на воде и волны.
Аноним 06/09/22 Втр 20:19:08 #102 №564785 
image.png
Каким образом в ДНК закодированы такие параметры организма как размеры и формы тела? Как клетки понимают в каком количестве им нужно делиться и в каком пространственном положении выставляться?
Аноним 06/09/22 Втр 20:43:15 #103 №564786 
image.png
>>564785
Единственное могу сказать чтоб менять размер там не сильно много генов нужно. Вот представители одного вида
Аноним 06/09/22 Втр 21:26:42 #104 №564787 
>>564785
Дакинза наверни, он это годно разжёвывает
Аноним 06/09/22 Втр 23:01:09 #105 №564790 
>>564726
Нисколько, для индустриальной революции нужен сначала легкодоступные уголь и нефть, а они кончились.
Аноним 06/09/22 Втр 23:22:43 #106 №564792 
Помню читал текст про бразильских студентов, которые могли пересказать труды платона или какого-то другого античного философа в оригинале, но при это не понимали текста сказанного. Может помнит кто?
Аноним 06/09/22 Втр 23:31:56 #107 №564793 
>>564792
Нахуя? Ты ж и так помнишьвсё что надо об этом. Нахер свой мозг всяким говном засираешь, пустыми подробностями.
Аноним 06/09/22 Втр 23:40:52 #108 №564794 
>>564793
Мне казалось, что это одна из мыслей текста, разве нет?

Кстати, никто не заметил, что гугл стал хуже искать, чем года 3-4 назад? Раньше по ключевым словам находил все что нужно, а сейчас одно говно кликбейтное в выдаче.
Аноним 06/09/22 Втр 23:47:45 #109 №564795 
videoplayback (1).mp4
Помогает ли шуба согреться в данной ситуации или, наоборот, только мешает (препятствует притоку тепла от камина)?
Аноним 06/09/22 Втр 23:51:15 #110 №564796 
>>564795
Если шуба уже теплая, потому что находилась в помещении, то поможет, если нет то нет.
Аноним 06/09/22 Втр 23:51:28 #111 №564797 
>>564795
Сложно, надо считать. Так-то шуба тоже нагревается снаружи, и это тоже сдвигает тепловое равновесие и мешает теплу от тела уходить наружу.
Аноним 07/09/22 Срд 07:20:05 #112 №564804 
Skrin.PNG
skrin2.png
>>564437 (OP)
Почему учебники Киселёва считаются нормальными, а учебники, скажем, Колмогорова - нет?
Связано ли это только с тем, что Киселёв разжёвывает вещи "интуитивно" и для полных имбецилов, в то время, как условный Колмогоров практикует более жёсткий и формальный стиль преподавания?

Просто я наткнулся на срач на тему того, что "саффки разрушили преподавание матана, потому что Киселёв видел УЧЕНИКА" и у меня сгорело седалище. Нет, у меня реально сгорел пукан от пикрилейтедов, особенно когда один быдлокодер прислал второй скрин, как пример ХОРОШЕГО, годного доказательства.

Может, я в чём-то неправ, но на мой взгляд оба пикрила - это же пиздец для совсем тупорылых имбецилов, которые
а) Без слова дядьки ничего не могут сделать и сами найти
б) Способны решать возникшие задачи исключительно армейским способом без применения аналитики.
Аноним 07/09/22 Срд 12:16:55 #113 №564816 
>>564437 (OP)
На чём в 2к22-м хранят инфу, нынче? Заебали харды, сыпятся, и хуй сбросишь SMART на некоторых дисках. Куча BAD-секторов. Кто-то понтуется SSD, типа могу сибе позволить ко-ко-ко, а там блядь 100000 циклов перезаписи, потом сыпется хард.
Есть какие-нить более заебатые ДНК-флешки, или какие-нить ёба-долговечные носители инфы, наподобие "вечного диска" стекляного, чтобы блядь по несколько миллиардов лет инфа могла храниться?
Разумеется, "вечный диск" записывать лазерами, в специальной лаборатории - нет резона, потому что это ебала пиздец.
Чё-нить более юзабельное изобрели уже, или токо хард с его обоссанных 2 года гарантии?
Может сразу в IPFS лить бекапы зашифрованные, или на торренты? Дык а кто сидировать будет, тогда?
Аноним 07/09/22 Срд 12:23:03 #114 №564818 
image.png
>>564783
А если свет узконаправленный? Лазерный луч, например?
Увидишь полосу?

А потом, хуле камешки, волна со скоростью которой ты сам движешься как тебя догонит то?
Аноним 07/09/22 Срд 13:23:11 #115 №564822 
>>564804
Объяснения базовых вещей и должно быть как для тупых. Аналитика набивается через решение задач требующих комбинации нескольких формул.
Аноним 07/09/22 Срд 13:24:20 #116 №564823 
>>564816
>Есть какие-нить более заебатые ДНК-флешки, или какие-нить ёба-долговечные носители инфы, наподобие "вечного диска" стекляного, чтобы блядь по несколько миллиардов лет инфа могла храниться?
Оптические диски в нормальных футлярах, пока не поцарапаешь инфа не проебется.
Аноним 07/09/22 Срд 15:52:16 #117 №564827 
>>564795
то что он в снегу и сверху натягивает шкуру большая ошибка
Аноним 07/09/22 Срд 16:03:59 #118 №564828 
а можно ли из говна и палок соорудить дома простейшие ИС?
Допустим я найду кремниевую пластину на авито, легированную бором, ебану на нее две полоски тут хуй знает,
Р2О5 подойдет для тепловой диффузии
, нагрею ее и буду ждать. Выйдет ли у меня транзистор, или где-то я обьебался? Или есть еще проще способ?
Аноним 07/09/22 Срд 16:15:18 #119 №564829 
>>564828
Таки можно, но ты у тебя должны быть сверхпрямые руки и куча расходников + оборудование, собрать которое само по себе челенжд.
Даже если ты собрался грубо послойно напылять транзисторы, то тебе нужна установка ультравысокого вакуума, а такая вещь вещь даже не в каждой лаборатории есть.
Аноним 07/09/22 Срд 16:24:08 #120 №564832 
>>564829
Дык нет, понятно что для напыления в вакууме оборудование нужно. Но мне же нужен, допустим, один транзистор, соответственно, два p-n перехода на большой пластине кремния, чтобы микроны с нанометрами не ловить. Если я два произвольных места на кремниевой пластине легирую фосфором, то у меня же будет два перехода? Или для легирование даже диффузионным способом тоже придется оборудование доставать? нагреть до 900цельсия я смогу наверное
Аноним 07/09/22 Срд 16:31:30 #121 №564833 
Можно ли считать, что современное общество выебало математику, ведь по статистике должно быть нормальное распределение, и раньше такое было в оценках. Типа если средняя оценка 7, то большинство оценок будет 6-8, остальных меньше, 0-2 минимум. А сейчас средняя оценка 7 значит, грубо говоря, 65% десяток, 25% единиц и немножко остальных оценок. И "колокол" распределения сосёт хуй.
Аноним 07/09/22 Срд 16:43:02 #122 №564834 
>>564833
Это хуета, училки ставят что хотят, не объективно, плюс твои данные говно а сам ты тупой школьник наверное.
Вангую если взять вообще все оценки которые ставят все училки, как раз будет нормальное распределение.

Ну и опять же, есть процессы стремящиеся к крайностям а не к среднему, а вот крайние пики соединяются уже каким-то тоже законом особым, это всё тоже есть в математике.
Аноним 07/09/22 Срд 16:46:18 #123 №564836 
>>564822
>Объяснения базовых вещей и должно быть как для тупых.
А не приводит это к тому, что челик привыкает всё решать через брутфорс и "ну это очевидно жи"? И, собственно говоря, именно умение анализировать поставленную задачу атрофируется?

То есть (очень грубый пример), такому челу, например, проще будет пересчитать в ручную все возможные варианты чем применить индукцию, ибо это думать надо, делать какие-то выводы, кококо никто не объяснил и не разжевал?
Аноним 07/09/22 Срд 16:54:41 #124 №564837 
>>564834
Какие училки, речь про оценки играм, фильмам и т.п.
Аноним 07/09/22 Срд 16:56:59 #125 №564838 
>>564833
>Можно ли считать, что современное общество выебало математику, ведь по статистике должно быть нормальное распределение
Дроч на "нормальное распределение" приводит к такому абортажу педагогической мысли, как "нормализация оценок" для подгона под эту белл курву.

> Типа если средняя оценка 7, то большинство оценок будет 6-8, остальных меньше, 0-2 минимум. А сейчас средняя оценка 7 значит, грубо говоря, 65% десяток, 25% единиц и немножко остальных оценок. И "колокол" распределения сосёт хуй.
Школьная программа, практически вся, проходится на уровне 7-8 просто без запарок, если ты тупо хоть немножко проявляешь внимание и умеешь думать. Даже если домашние задания делаешь за пять минут до урока.
Если же думать ты не умеешь, то сосёшь в математике практически всегда по ВСЕМ фронтам.

Когда я учился, в принципе, так в моём классе и было. Либо чел что-то знает и вывозит на 7 и выше, либо чел сосет и получают неуд.
Аналогично в универе на мат-логике и алгебре и матане было, из-за чего они считались "фильтрующими предметами". На моём потоке этот колокол вообще был вверх ногами - средних оценок было меньше всего, ибо в потоке были либо задроты с математических школ и подобные, либо будущие кодомакаки, которых либо отфильтровал сам матан, как таковой, либо недостаточное усвоение материала по другим курсам.
Аноним 07/09/22 Срд 17:01:31 #126 №564839 
>>564838
Да бля, откуда у вас эти проекции на школу? Еще и с 10 баллами? Речь про имдб и метакритики всякие, сука.
Аноним 07/09/22 Срд 17:34:31 #127 №564840 
>>564836
Ну так я же скозал, объяснения для тупых, а решение задач уже с применением аналитики и полученных знаний.
Аноним 07/09/22 Срд 17:36:42 #128 №564841 
>>564839
А кто сказал, что распределение ИРЛ в принципе должно быть нормальным?
Аноним 07/09/22 Срд 17:41:11 #129 №564842 
>>564841
А если не ирл, то где? В фантазиях математиков?
Аноним 07/09/22 Срд 17:53:07 #130 №564844 
>>564842
Для определенных явлений оно работает, для определенных нет, оно не должно быть везде.
Аноним 07/09/22 Срд 18:18:19 #131 №564845 
>>564843
А надо бы выучить.
Электроника это ж прикладная физика.
Аноним 07/09/22 Срд 18:25:54 #132 №564847 
>>564844
Каких явлений, если ирл не работает?
Аноним 07/09/22 Срд 18:31:54 #133 №564848 
>>564847
Она работает для определенных вещей, в той вики можешь посмотреть для каких именно. Я хз откуда взялось мнение про то что нормальное распределение должно быть везде.
Аноним 07/09/22 Срд 18:40:59 #134 №564849 
>>564847
Возьми половину ветви нормального распределения, приложи к пикам, и увидишь что оно почти везде и есть.
Пик-то необязательно в среднем значении.
Аноним 07/09/22 Срд 19:17:18 #135 №564852 
>>564843
ЭДС/мощность и внутреннее сопротивление.
Аноним 07/09/22 Срд 19:24:53 #136 №564853 
>>564850
Напряжение, ток, точность выдаваемого напряжения, шумность-сколько отклонений и с какой частотой отклоняется от заданного значения, форма сигнала этих помех, помехи при разной нагрузке, если регулируемый-диапозон регулирования, причём при разном выдаваемом напряжении может иметь возможность выдавать разный максимальный ток, помехи при разном выдаваемом напряжении если регулируемый.
Проседание напряжения под нагрузкой, наличие защит от коротких замыканий, обратного напряжения
Обычно заморачиваться вообще не имеет смысла, но мало ли какие задачи
Бывает что +-5% на всё и пофиг, а бывает что если +-0.005% хотябы в одном из перечисленных параметров и измерения не будут получаться.
Аноним 08/09/22 Чтв 04:22:52 #137 №564875 
>>564823
Та не, какие нафиг оптические диски, они же реально быстро царапаются даже от вращения в говенном дисководе. Или пылинка попадёт на поверхность, протираешь тряпочкой - сразу видно царапины на поверхности. А биты настолько маленькие, что сразу несколько бит перекрывает эта маленькая царапина, и уже нихуя не читается.
Поэтому я думал, почему же не сделали стекляные оптические диски, с хорошим, качественным стеклом, наподобие Gorilla Glass, которое на экранах смартфонов пришпандоривают, что оно не бьётся и не царапается даже.
Также, я где-то видел самовосстанавливающиеся стёкла, где со временем царапины попросту исчезают.

Но диски это такая хуйня... Алюминиевая плёночка, блядь. У алюминия температура плавления 600 цельсия где-то, стоит разгореться пожару и пизда инфе.

Надо что-то реально неубиваемое придумать, чтобы и перезаписывать можно было, и чтобы инфа хранилась по несколько лярдов лет.

Какие-то прочные материалы, наподобие этой смеси: https://itc.ua/news/uchenye-sozdali-vechnyj-mylnyj-puzyr-iz-plastikovyh-chasticz-gliczerina-i-vody-on-prosushhestvoval-rekordnye-465-dnej/

или такой вот кварцевый диск на 360 терабайт:
https://itc.ua/news/issledovateli-sozdali-prakticheski-vechnyiy-360-terabaytnyiy-disk-iz-kvartsa/

Но такую хуйню записывать просто так не получится.

А пока, только HDD, SSD, флешки, ну и оптические диски эти - болванки...
Аноним 08/09/22 Чтв 06:20:08 #138 №564876 
>>564816
>На чём в 2к22-м хранят инфу, нынче? Заебали харды, сыпятся, и хуй сбросишь SMART на некоторых дисках. Куча BAD-секторов.

Voyager 1 сейчас в 22 световых часах от Солнца. Его пекарня работает уже 45 лет, она пережила радиационные пояса Земли, десятки вспышек на Солнце, пересекла гелиопаузу и ей норм.

Управление Вояджером осуществляют три компьютерные системы. Эти системы можно перепрограммировать с Земли, что позволяло менять научную программу и обходить возникающие неисправности. Основную роль играет командная компьютерная подсистема (англ. computer command subsystem), содержащая два независимых блока оперативной памяти по 4096 машинных слов и два процессора, которые могут работать как дублируя друг друга, так и независимо. Ёмкость запоминающего устройства на основе магнитной ленты — 536 мегабит.
Аноним 08/09/22 Чтв 08:01:34 #139 №564879 
Можно ли сделать брелок-индикатор магнитного поля? Эдакая брошь для космонавтов для работы в зоне около радиационного пояса, чтобы не залетали куда не надо

Продолжая это вопрос, а будет ли работать кольцо-индикатор поля, надетое на палец? Будут ли малые токи человека мешать?

А если человек с брошью/кольцом индикатором находится внутри электрозащищённой зоны - внутри клетки Фарадея - бедут ли такой сигнализатор сигнализировать?
Аноним 08/09/22 Чтв 11:02:48 #140 №564884 
>>564875
> Также, я где-то видел самовосстанавливающиеся стёкла, где со временем царапины попросту исчезают.


Интересно, как это сделано..?
Аноним 08/09/22 Чтв 13:06:39 #141 №564896 
>>564437 (OP)
Сколько подъёмной силы полиэстерных крыльев (2шт, Л и П, 2х8 метров каждое) нужно, чтобы поднять чела 80кг в воздух?

Сколько взмахов в секунду?

У меня есть идея летающего костюма с крыльями. Нужны большие крылья по бокам, много систем безопасного приземления, и чтобы при поломке крыльев они ломались частично а не полнось и сохраняли работосопсобность.
.
Я так думаю, плотность энергии жидкого топлива очень высокая, в 2 куб. дециметрах умещается много. Т.е. 2 бака по 1 л каждый на поясе дадут тебе несколько КвТч энергии.
Нужен мини-двс на спину или живот, из современных сплавлов, максимально легкий. Он будет питать моторчики, а они регулироваь крылья. Поршень крутить мини-коленвал, а он - махать крылья в диапазоне +-30 градусов(при взлете - +15..+50 градусов от горзонтали, чтобы не разбить об твердую поврехность)

Самый кайф будет в небе. там можно будет ловить потоки воздуха и планировать, как орёл.


И вот тут вопрос - какие крылья нужны, чтобы планировать как орел, учитывая, что распределние веса данной летающей конструкции будет типа 100кг в центре и крылья по 8 кг каждое?

Материал крыльев - ткань или полимеры гибкие, очень тонкие. Каркас углепластиик/армированный платсик, наверное
Аноним 08/09/22 Чтв 15:09:04 #142 №564919 
Чем является запах "свежескошенной травы"? Это хлорофилл, или может соки какие испаряются? Если во дворе нарвать травы и взбить в миксере, получится ли он?
Аноним 08/09/22 Чтв 15:23:05 #143 №564921 
Можно ли получить потомство от рандомной клетки человека, чтоб пездюк был неотличим от полученного от обычной ебли? А от эмбриона, его стволовых клеток? Ввести его гены в другую яйцеклетку, чтоб там всё стандартно рекомбинировало.
Аноним 08/09/22 Чтв 15:24:41 #144 №564922 
>>564919
Нагуглил за тебя, пидр: https://hij.ru/read/5393/
Аноним 08/09/22 Чтв 15:45:41 #145 №564924 
>>564922
Благодарю. Мог бы и сюда скопипастить, пидр.
Аноним 08/09/22 Чтв 16:06:24 #146 №564925 
16624990710920.jpg
>>564896
>Самый кайф будет в небе. там можно будет ловить потоки воздуха и планировать, как орёл.
>И вот тут вопрос - какие крылья нужны, чтобы планировать как орел
Аноним 09/09/22 Птн 04:05:48 #147 №564951 
>>564437 (OP)
Помогач, хочу сковородку!
Чтобы она служила лет 50, и чтобы нихуя на ней не пригорало.
Чтобы она не плавилась, блядь, когда на костре жаришь чо-то,
и чтобы там не собирался пятидесятилетний нагар.
Чтобы она была крепкой, и чтобы не нужно было рихтовать вмятины на её днище, и чтобы было наилучшее антипригарное покрытие, хуй знает не тефлон - это точно. Какой-нибудь гранит, или мрамор, наверное.
Видел полностью гранитные сковородки, но если такая пизданётся об пол - она треснет пополам, нахуй.
Надо чё-то попижже. Чтобы это был не алюминий, или ёбанный силумин. Чтобы это был или чугун или нержавейка, или хуй знает что ещё.
Чтобы ручка была съёмная, чтобы её можно было в духовку засунуть, и там жарить чо хочешь.
И главное, чтобы это была наилучшая сковородка по критерию цена-качество. Желательно, чтобы там знак качества, советский, стоял, как на моём утюге, который уже 60 лет работает и не выёбывается.

Короче хз что взять. Фирм дохуя, и каждый кулик своё болото хвалит. Мне надо на века чтобы было, как реликвия, для потомков. Чо скажешь?
Аноним 09/09/22 Птн 04:40:12 #148 №564952 
>>564774
Да, т.к. геометрия там не как у Евклида, а специально сделанная так, чтобы во всех системах отсчета скорость света это скорость света.
Квантовая механика Аноним 09/09/22 Птн 05:15:12 #149 №564953 
1. Вот электрон понимают как энергетическое "облако" вокруг ядра атома, где он то ли быстро перемещается по нему, то ли одновременно находится везде, то ли размазан по всему. А как электрон выглядит в отрыве от атома? Когда он сам по себе, он такое же вероятностное облако его нахождения? Какая будет форма и размер у этого облака, если на него ничего не действует, в смысле он не находится в электромагнитном поле и прочее? Когда говорят о движении электрона, подразумевается движение "центра масс" этого облака?

2. Фотон это тоже вероятностное облако его обнаружения? А еще он фотон-частица и фотон-волна. Как все это можно собрать вместе.

3. В описании опытов (например с двумя щелями где электрон ведет себя то как частица, если его обнаруживают, через какую щель он пролетел, то как волна - если нет) часто говорится в стиле, что источник испускает по одному электрону, за которым потом наблюдают. Не понятно, как чисто технически это возможно (особенно в 10-е 20-е годы прошлого века), что будет испускаться именно по одному электрону, если электрон это такая мутная вещь, которую обнаруживаешь только провзаимодействовав с ней другими частицами. Ну и просто это кажется чем-то максимально сложным, если не невозможным, что ты можешь уверенно утверждать что например каждые полсекунды от вещества отлетает только один электрон. Мне кажется в реальности там всегда пучок или луч электронов, а от этого интерпретации опытов должны быть/могут быть другими.
Аноним 09/09/22 Птн 05:34:34 #150 №564954 
>>564953
Я недавно углубился в эту тему, и пока пришёл к такому выводу что тут главный вопрос это понять какие эксперименты и понять как доказывают что не существует объективной реальности.
Насколько понял это БАЗА квантовой механики, она типа постулирует что нету, не существует объективной реальности, есть только информация у конкретного наблюдателя. У другого наблюдателя будет другая информация и соответственно другая реальность. Ну, может быть, да и не будет на 100% совсем уж одинаковой.
Причём постулирует квантовая механика это не с религиозно-верунской позиции что типа "наш мир в принципе непознаваем полностью", а именно есть несколько каких-то экспериментов, которые доказывают отсутствие объективной реальности. Т.е. буквально что луны не существует пока ты на неё не смотришь. Ну это утрированно, но какой-то похожий более реальный пример я придумывал но сейчас забыл, думал его привести как аргумент "против" адептам квантмеха, жаль вылетел из головы.
Аноним 09/09/22 Птн 05:36:43 #151 №564955 
>>564951
>Чо скажешь?
Что лучше тебе сдохнуть, советское одебилевшее совкодрочащее говно.
Аноним 09/09/22 Птн 05:43:52 #152 №564956 
>>564953
Электрон облако вращается вокруг атома как сильно скрюченное кольцо. Другие электроны, и его собственное тело, закрывают для него некоторые пути. Когда электрон схлопывается в точку, старое облако исчезает, а новое начинает распространяться из точки в границах старого.
Оставшийся в одиночестве электрон, предположительно, будет расти, пока не найдет с чем ему прореагировать, и не схлопнется.
Если есть возможность подсчитать количество атомов в теле, будет и возможность посчитать электроны.
Аноним 09/09/22 Птн 05:45:02 #153 №564957 
>>564954
>она типа постулирует что нету, не существует объективной реальности, есть только информация у конкретного наблюдателя
нет
Аноним 09/09/22 Птн 05:47:02 #154 №564958 
>>564954
>луны не существует пока ты на неё не смотришь
Нет. Пока в нее продолжают стукаться фотоны, она находится под "наблюдением". Магия возможна только в истинной тьме.
Аноним 09/09/22 Птн 05:51:45 #155 №564959 
>>564957
Я это почерпнул с канала световой конус(по английски).
Рассказывает он конечно не очень, фриков обозревает вообще неубедительно, но сам преподаёт насколько понял, плюс во многих видосах тупо даёт информацию тупо из заграничных учебников собственно главных авторов квантовой механики, которую её непоколебимую базу и сделали, не добавляя в эту информацию "своё видение", тупо что в учебниках основателей, то и говорит.
И про отсутствие объективной реальности он говорил точно, и не раз, совсем недвусмысленно а конкретно, типа это одна из баз квантовой механики, чуть ли не основная. И что самое удивительное для меня, не постулируется допущением, а доказывается экспериментами. Насколько понял если этого бы не было, сама квантовая механика бы не работала, там бы вылазило куча логических противоречий.
Аноним 09/09/22 Птн 05:54:05 #156 №564960 
>>564959
>доказывается экспериментами
Скажи, Анон, а эти эксперименты, они сейчас с тобой в одной комнате?
Аноним 09/09/22 Птн 05:57:22 #157 №564961 
>>564958
Да, я когда набирал то вроде вспомнил какой контраргумент придумал, и вроде это оно.
Типа всегда есть взаимодействие с полями, они везде, а потом в любом случае, хоть и сильно косвенно, через множественные "переотражения" взаимодействий передаёт информацию о состояние частицы всем, значит просто невозможно такое что типа "никто не смотрит".

Ещё пока это набирал, подумал что вроде ещё тогда думал о статистике. Типа какой смысл говорить что "не существует", если можно со 100% вероятностью предсказать что он там будет. Типа шар в коробке, его нет при закрытой крышке, но если ты видел его когда она была открыта, то когда откроешь он 100% опять там будет. Или результат заметный его распада, но он не исчезнет прям совсем и это со 100% вероятностью.
Так что тупо об этом говорить.
А раз весь мир квантовомеханический, то рразэто работает на макрошаре, работает и для любых квантовых частиц.
Аноним 09/09/22 Птн 05:58:23 #158 №564962 
>>564960
Я в них пока не поверил и не вникал в них особо.
Можешь посмотреть описание экспериментов на его канале.
Аноним 09/09/22 Птн 06:05:43 #159 №564963 
>>564960
https://youtu.be/SWut6LF46Kw

Видео не то, сам ищи, просто как ссылка на канал.
Аноним 09/09/22 Птн 06:36:04 #160 №564964 
Почему парадокс лжеца существует? Если попытаться прочитать и вопринять фразу «Данное утверждение ложно» возникает проблема, поскольку начало фразы, а именно слова «Данное утверждение» ссылаются сами на себя и таким образом получается что-то вроде бесконечного цикла «Данное утверждение «Данное утверждение «Данное утверждение»»» и так далее и таким образом невозможно дочитать до слова «ложно». То есть фразу «Данное утверждение ложно» по сути воспринять вообще невозможно. Это по сути то же самое что пытаться понять или воспринять фразу «Птица картофель». Смысл отсутствует. Где я не прав? Почему данное выше рассуждение не может являться решением парадока?
Аноним 09/09/22 Птн 07:10:26 #161 №564965 
>>564964
> Почему данное выше рассуждение не может являться решением парадока?

Потому что ты прицепился морфологии фразы, а не к ее смыслу.
Используй фразу «Я лгу».
Аноним 09/09/22 Птн 07:47:51 #162 №564968 
>>564960
https://youtu.be/P3daKHQO9iU

Ютуб подсунул опять.

И в этом видео он говорит ещё об одном прошлом видео, в котором тоже про отсутствие объективной реальности, видимо на эту тему и больше вообще видео на канале чем 2.
Аноним 09/09/22 Птн 08:19:14 #163 №564971 
>>564955
Ебало завали, червь. А то на моей сковороде чугунной появится ещё одна вмятина, как только попадёшься мне на глаза, подхуйловский предатель-говнарь.
Аноним 09/09/22 Птн 11:23:32 #164 №564974 
Бля, что-то про импульс хотел спросить но забыл(
Аноним 09/09/22 Птн 11:37:05 #165 №564975 
>>564925
Я абсолютно серьёзен. Выеди(почему красным подчеркивается?) за город, посмотри на хищных птиц. Они бреют в небе, очень редко делая взмахи. Ловят потоки воздуха и кружат над территорией.

Я так же хочу летать, но я чел.
Аноним 09/09/22 Птн 12:05:32 #166 №564976 
>>564975
Закон квадрата-куба, чел.
Причём по несущей способности воздуха легко сделать нужный размер крыльев для массы человека. Допустим птица весом 1кг хорошо летающая, а не как курица, вполне небольшая, достаточно всего лишь пропорционально весу увеличить площадь крыльев, а т.к. площадь растёт в квадрате, то вполне небольших крыльев достаточно бы было, но...
Сделать маленькое прочным легче чем сделать большое прочным, при увеличении крыльев площадь будет увеличиваться в квадрате, но вес в кубе, а чтобы этот дополнительный вес относительно "пропорционального"(линейного типа, в реале он неленейный, поэтому записал как "дополнительный" веса крыльев, они выдерживали, нужно делать их прочнее, а это ещё вес, ещё весь-нужно делать ещё прочнее, плюс больше площадь надо делать, т.к. вес добавился, а раз площадь добавилась, значит ещё прочнее надо делать, а это ещё вес... и ещё добавлять площадь и ещё прочнее надо делать, и ещё вес и ещё площадь...
И оно так накапливается и накапливается очень сильно и быстро.
Поэтому у больших птиц крылья это может быть 90% их веса, а у мухи тело жирное, а крылья весят 1% от её веса.
У тебя же с весом 80кг крылья будут весить тонну, а веся тонну такое устройство будет неуклюжим полюбому.
Аноним 09/09/22 Птн 13:22:50 #167 №564977 
>>564976
>крылья будут весить тонну
углепластиковый каркас + ткань
Аноним 09/09/22 Птн 14:12:16 #168 №564979 
>>564977
Ну да, будешь делать всё сложнее и дороже, это помогает, но несильно, кубическая степень сильнее всего во вселенной, она чёрные дыры создаёт, а ты какой-то углепластик бля. Ткань, лол
Всё уже давно опробовано, если бы было можно так летать, давно бы сделали и летали.
SR-71 создан в 1960м, уже после войны практически самой ебовейшие самолёты создавали, а ты думаешь какую-то махалку для человека не сделали бы. Это ж очень перспективно, для каждого солдата, лёгкая и суперманёвренная штука, каждый солдат становится воздушным независимым десатником, плюс самолётом-штурмовиком, быстрые налёты, быстрое десантирование, перемещение, быстрое отступление, летать между деревьев кустов как птица, недоступная цель для медленно поворачивающих ракет. Уже бы давно сделали будь возможно.
Аноним 09/09/22 Птн 14:46:40 #169 №564980 
>>564979
я не про армию

это ебанина 17х3 метра, можно сказать дельтоплан/параплан с машущими крыльями, который крепится к человеку.
для армии слишком немобильная и неудобная, она для кайфовых полетов над полями
Аноним 09/09/22 Птн 14:57:51 #170 №564982 
>>564980
Ну так сделай такое же лёгкое крыло как жёсткое делают, и при этом махающее, лол, такого же веса.
Аноним 09/09/22 Птн 17:50:15 #171 №564984 
>>564980
>это ебанина 17х3 метра
А кстати хули такая большая? Сделать крыло можно ну 10кг весом.
Площадь крыла птицы весом 1кг, ну хз, явно сильно меньше метра. 0.2-0.3 метра квадратного.
х90=18-27 квадратов, 9х2, 9х3
И должен уже вес 90кг общей конструкции неплохо так плавненько планировать с крылом такой площади.
Аноним 09/09/22 Птн 20:12:22 #172 №564987 
>>564984
Ну я же говорю что это возможно. По сути дельтаплан, надевающийся вокруг пилота, но с двигателем для махания крыльями. Вопрос в его весе на КПД. И в частоте махания для взлета/поддержания высоте вне потоков воздуха


Как вариант, реактивный движок, но а) оч шумно б) нужны регулируемые сопла и проч. для плавного взлета и посадки = сложно
Аноним 10/09/22 Суб 14:37:30 #173 №564992 
>>564951
Бампецкий, а то тут ещё вилки крепкие, надо будет выбрать тоже, а то когда вилкой в глаз по пьяне тычу гостям - иногда зубчики, блядь ломаются, и в середине, под плёнкой нержавейки - видно дешёвый силумин.
Аноним 10/09/22 Суб 15:30:45 #174 №564993 
Какие интернет-ресурсы вы читаете по тематике сабжа?
Есть какие-то русскоязычные форумы (не мертвые)?
Аноним 10/09/22 Суб 18:36:06 #175 №564997 
IMG2032.MP4
>>564993
смотрю научпоп ролики в телеге на хохляком канале, хотя не понимаю 90% текста там
Аноним 13/09/22 Втр 22:14:46 #176 №565064 
>>564437 (OP)
В новуке реально рассматриваются такие понятия как бесконечное время или это какая-то абстракция?
>В КЦК Вселенная проходит через бесконечные циклы, где в каждом предшествующем цикле (эоне) время в будущем стремится к бесконечности, что оказывается сингулярностью Большого взрыва для следующего
https://ru.wikipedia.org/wiki/Конформная_циклическая_космология
Аноним 13/09/22 Втр 23:33:07 #177 №565065 
>>565064
Время и есть абстракция.
А бесконечное время это какой то бредовый оксюморон.
Аноним 13/09/22 Втр 23:38:36 #178 №565066 
>>565065
Вот смотри. "Нет ничего кроме частиц и их взаимодействий."
Допустим все взаимодействия описываются энергией кинетической mv^2/2 и импульсом mv.
Но скорость записать без времени.
Можно ли как-то выразить можно ли как-то импульс или кинетическую энергию записать без времени?
Аноним 13/09/22 Втр 23:39:43 #179 №565067 
>>565066
>Но скорость записать без времени.
*нельзя
Аноним 14/09/22 Срд 00:00:53 #180 №565068 
>>565066
Числа и формулы это тоже абстракции в мире ума.
Соответственно абстракция время может существовать в формуле.
Аноним 14/09/22 Срд 00:07:29 #181 №565069 
>>565068
Ну я тоже уже примерно знал ответ когда это писал.
Всё относительно. Относительно движения в других местах
А хотя... Бля, нет, допустим, все частицы во вселенной стали двигаться в 2 раза медленнее, в 2 частицы двигаются как и раньше. Энергии ведь выделится такое же количество при их столкновении как и раньше, это не относительная величина. Получается скорость абсолютна через энергию.
Она относительна только относительно частиц или которые провзаимедейсвуют с ней, или которые не провзаимеодйствуют, но чисто теоретически, до того как взаимодействие произойдёт. А когда оно произошло то всё нихуя не относительно относительно тех частиц которые не провзаимодейстовали с ней.
Время выводится через энергию получается как-то. Потому что если "замедлить все частицы во вселенной в 2 раза" кроме нескольких других, это будет не ровно то зе что и замедлить время в 2 раза. Будут отличия по энергии событий и процессов.
Аноним 14/09/22 Срд 00:18:33 #182 №565070 
Руководитель отдела ядерной планетологии Института космических исследований (ИКИ) РАН Игорь Митрофанов считает, что человечество обязательно вымрет, как вымерли когда-то динозавры, возможно, даже в течение ближайших 100 миллионов лет.

Ученый РАН: человечество может вымереть в ближайшие сто миллионов лет
«В будущем Земли будут такие столкновения с крупными астероидами, которые земная цивилизация не переживет», – сказал Митрофанов на научном семинаре в Москве. Он уточнил, что, возможно, на Земле произойдет нечто подобное тому, что когда-то «не пережили динозавры». При этом ученый добавил, что подобные события могут случаться достаточно часто, примерно каждые 100 миллионов лет.


Мнение научного сообщества двача, нам действительно хана?
Аноним 14/09/22 Срд 00:19:30 #183 №565071 
>>565069
> все частицы во вселенной стали двигаться в 2 раза медленнее, в 2 частицы двигаются как и раньше. Энергии ведь выделится такое же количество при их столкновении как и раньше

Если частицы двигаются медленнее, значит они получили меньше энергии для движения. Соответственно при их столкновении выделится меньше энергии.
Аноним 14/09/22 Срд 00:23:42 #184 №565072 
>>565070
> Он уточнил, что, возможно, на Земле произойдет нечто подобное тому, что когда-то «не пережили динозавры».

То что «не пережили динозавры» таки пережили приматы.
А вот то что через 5млрд лет потухнет солнце и если человеки к этому времени не сьебут с земли, то им кирдык, это факт.
Аноним 14/09/22 Срд 00:57:49 #185 №565073 
>>565066
>Можно ли как-то выразить можно ли как-то импульс или кинетическую энергию записать без времени?
Нет, совсем никак.
В физике есть фундаментальный принцип стационарного действия, который сводиться к занулению вариации некоторого интеграла по времени. Без формализации времени как некоторой величины невозможно формулировать энергию и импульс вообще.
Аноним 14/09/22 Срд 01:17:17 #186 №565074 
>>565073
Но ведь интегралу похуй по какой области считать.
Он что по времени считает, что по длине, что по тупо цифрам безразмерным. "Интеграл от 0 до t", "Интеграл от 0 до 5".
Время откуда выводиться должно.
Что насчёт того что я писал там типа время как-то из энергии должно выводиться?
Аноним 14/09/22 Срд 01:37:26 #187 №565075 
>>565072
>То что «не пережили динозавры» таки пережили приматы.
В соответствии с палеонтологическими данными, наиболее древние приматы (представители рода Plesiadapis) известны с позднего палеоцена, 55—58 млн лет назад.
Там приматами и не пахло даже
Аноним 14/09/22 Срд 02:25:25 #188 №565076 
>>565075
Не суть. Смысл был в том, что динозавры вымерли так как не приспособились к изменившимся условиям окружающей среды в связи с падением астероида. При повторении подобного человечество поредеет конечно, но не вымрет точно.
Аноним 14/09/22 Срд 03:03:47 #189 №565077 
>>565076
> динозавры вымерли так как не приспособились
Не все. Повымирала мегафауна и вообще все что крупнее енота.
>При повторении подобного
Это исключено
Аноним 14/09/22 Срд 12:04:41 #190 №565085 
>>565077
>Это исключено
Ты шизик-верун, ничего не исключено.
Аноним 14/09/22 Срд 14:40:38 #191 №565092 
>>565085
Атомной бомбой взорвут, я в кино видел
Аноним 14/09/22 Срд 15:03:39 #192 №565095 
Человек получает энергию внутри своего тела условно говоря сжигая водород?
Т.е. с пищей и водой кроме самой собственно H2O которая в них содержится, в составе органических молекул пищи есть тоже много H, выделяет ли человек больше воды чем изначально в виде уже цельного соединения H2O поглащает?
Аноним 14/09/22 Срд 15:50:32 #193 №565096 
>>565085
>придумывает детский вариант событий
>Ты шизик-верун
завязывай смотреть лунтика и поней, открой для себя мир, а то так и останешься ботом с односложными ответами
Аноним 14/09/22 Срд 16:14:54 #194 №565097 
>>565096
Какой мир, дегер? Речь была о катаклизмах схожем с тем что привело к вымиранию динозавров, крупные астероиды падают постоянно с определённой периодичностью, супервулканы извергаются тоже постоянно с определённым циклом, ледниковые периоды случаются и очень сильные. Ты шизик.
Аноним 14/09/22 Срд 16:20:22 #195 №565099 
>>565097
> дегер
дальше не читал малолетку обоссанную
Аноним 14/09/22 Срд 16:32:54 #196 №565100 
>>565099
Смешно ты порвалась дурочка тупая.
Аноним 14/09/22 Срд 16:52:04 #197 №565101 
o4tzCv5RLvY9-port.jpg
Предположим, что мне удалось достать немного какуль Путина из секретного чемоданчика, куда их складывают его охранники.
Смогу я узнать какими-то анализами, что ел вождь пару дней назад?
Аноним 14/09/22 Срд 17:52:40 #198 №565106 
Screenshot 2022-09-14 at 19-45-34 nNpEX22zPCE.jpg (JPEG Image 1440 × 1920 pixels) — Scaled (31).png
Screenshot 2022-09-14 at 19-51-43 bKQsaEsrxAI.jpg (JPEG Image 445 × 500 pixels).png
>>564437 (OP)
Насчет формы спинки носа, бывают, в основном вогнутая (пик 1), прямая и выпуклая.

Вопрос про выпуклую:

Я заметил, что у некоторых людей в детстве вогнутая спинка носа, но с возрастом она превращается в прямую. Можно ли в таком случае считать вогнутую спинку носа - инфантильной или педоморфной чертой, а выпуклую - максимально зрелой (пераморфной)?

Ну и на том же рисунке Конрада Лоренца "Baby schema" (пик 2) у ребенка вогнутая спинка, а у взрослого выпуклая. Т.е. видимо такие мысли у биологов были уже давно.

===
Собственно с пигментацией все практически так-же, многие дети в детстве имеют светлую пигментацию, но потом "дозревают" до темной.
Электронтка Аноним 14/09/22 Срд 19:21:32 #199 №565108 
Я тут дома нашёл солнечную панель небольшую, на бумажке написано что выдаёт максимум 9 вольт, 0,333 ампера, но когда я прозвонил мультиметром, то он показал мне на достаточно слабом освещении в помещении через окно 7,8 вольт 5 ампер! Вот есть люди шарящие, как это вообще могло произойти?
Аноним 14/09/22 Срд 19:42:05 #200 №565109 
ParamoreHardTimesOFFICIALVIDEO(1).gif
С каждым днем все опаснее?

И интеллект и личностная черта "добросовестность" подвергаются дисгенной фертильности в по-крайней мере некоторых европейских популяциях. Но генотипическое снижение интеллекта компенсировалось улучшением среды, но вроде с недавних пор этот положительный эффект закончился.

И интеллект и добросовестность обратно коррелируют с преступностью. Т.е. чем ниже у людей интеллект и/или добросовестность тем более вероятно он нарушит закон и станет преступником.
К слову преступники оставляют больше детей, следовательно "генов преступности" в таких популяциях становится больше.

Т.е. потенциальных преступников с каждым новым поколением становится все больше и больше.

Что вы думаете насчет этого?
Аноним 14/09/22 Срд 19:52:46 #201 №565110 
>>565109
>Что вы думаете насчет этого?
Загнивающий запад загнивает.
Аноним 14/09/22 Срд 19:59:54 #202 №565112 
>>565110
Ну западу далеко до стран СНГ :-)
Все-таки по уровню законопослушности они намного превосходят наши страны, та-же Дания, например, находится на 1 месте в мире по уровню законопослушности.
Аноним 14/09/22 Срд 20:36:35 #203 №565113 
>>565112
Да законопослушных хуесосов легко выебать, вырезать, а потом ещё раз выебать.
Аноним 14/09/22 Срд 20:38:51 #204 №565114 
>>565108
Одновременно и 7.8 вольт показал и 5 ампер?
Аноним 15/09/22 Чтв 07:07:19 #205 №565120 
Посоветуйте литературу про нервную систему, мозг: строение, принцип работы.
Аноним 15/09/22 Чтв 07:27:00 #206 №565121 
>>565114
Да.
Аноним 15/09/22 Чтв 11:11:24 #207 №565130 
>>565121
Т.е. у тебя два мультиметра и ты ОДНОВРЕМЕННО двумя измерял?
Аноним 15/09/22 Чтв 18:37:39 #208 №565153 
Нет, я измерял одним мультиметром и интервалом не более 20 секунд. Свет явно не менялся, а если и менялся то совсем чуть чуть
Аноним 15/09/22 Чтв 19:31:26 #209 №565156 
>>565153
Года измерял 7.8 вольт там не было 5 ампер, когда измерял 5 ампер там не было 7.8 вольт. Возьми два мультиметра и посмотри. Или возьми такой кусок проволоки, измеряя ток мультиметром, чтобы было 2.5 ампера, (подбирая длину куска проволоки), так ты сразу мультиметр цеплял, а так ещё с одной стороны между контактом и мультиметром проволку прикрепи, а потом эту же проволоку которая с мультиметром вместе давала ток 2.5 ампера, и ко второму контакту прикрепи убрав мультиметр, и так измерь напряжение мультиметром чтобы там та проволока при этом была тоже между контактами.
Знаешь хоть надеюсь когда последовательно нужно соединять а когда паралельно.
Аноним 16/09/22 Птн 07:27:18 #210 №565184 
Да, понял, спасибо. Когда как цеплять знаю
Аноним 16/09/22 Птн 16:03:07 #211 №565211 
>>564437 (OP)
Сап сайнач. Аноны, предложите свои аргументы в пользу антинаучности соционики - хочу узнать что думают анонимные ученые-мужи.

мимо не верю в соционику
Аноним 16/09/22 Птн 17:10:09 #212 №565215 
>>565211
Хуита же с методологией уровня гороскопов, а с развитием когнитивных наук классификация характеров вообще стала антинаучной.
Аноним 16/09/22 Птн 17:50:57 #213 №565217 
>>565211
>соционики
Там есть сбор статистики? Если да то вполне научные результаты может давать. Только я хз что за соционика, как и социология предсказывает поведение людей? В любом случае этот ответ универсальный, если есть сбор статистики, и всё делать правильно, вполне научные результаты может давать.
Аноним 16/09/22 Птн 17:51:35 #214 №565218 
>>565215
Психология например вполне научной может быть. Конечно часто нет, но в принципе может.
Аноним 16/09/22 Птн 18:01:36 #215 №565219 
Бля, а вот интересно, на солнечных панелях падение напряжения под нагрузкой. И есть какое-то сопротивление нагрузки, при которых снимаемая мощность максимальна, т.е. оптимальное соотношение выдаваемого напряжения и тока. И сопротивление подбирать легко.
Но мощность с солнечных панелей снимают инвертором. Так вот, может ли инвертор быть тем самым сопротивлением?
Типа солнечная панель это и не источник тока и не источник напряжения и не источник мощности. А инвертор может вытягивать определённую мощность, но ток и напряжение при этом чисто какие получатся такие получатся.
Хм, в принципе можно походу, только дофига отслеживаемых параметров должно быть и много регулировок работы, сложное устройство получается.
Аноним 16/09/22 Птн 18:37:22 #216 №565220 
>>565218
Где и когда она была научной?
Аноним 16/09/22 Птн 19:28:25 #217 №565222 
>>565219
Падение напряжения вызвано внутренним сопротивлением и специфическим для полупроводников порогом.
Мощность элемента больше считается по выдаваемому току, а не напряжению, в этом смысле чем меньше сопротивление нагрузки тем лучше. Но на практике сопротивление нагрузки должно быть не меньше внутреннего сопротивления, в противном случае вся мощность будет уходить на нагрев внутреннего резистора. А вообще должно быть больше в два-три раза в зависимости от элемента.
С инвертором все определяется буфером(аккумулятором) и выходным каскадом, инверторы автоматом задают нагрузки для элемента. По факту там стоит генератор тока + специальный стабилизатор для буфера.
Аноним 16/09/22 Птн 19:37:17 #218 №565225 
>>565220
В предсказании поведения масс и даже отдельных людей по статистическим данным.
Аноним 16/09/22 Птн 19:46:08 #219 №565226 
>>565225
Как-то неубедительно звучит
Аноним 16/09/22 Птн 20:01:54 #220 №565227 
>>565211
> Аноны, предложите свои аргументы в пользу антинаучности соционики - хочу узнать что думают анонимные ученые-мужи.

Ненаучна потому, что не имеет предсказательной силы. Психотипы не жёстко установлены, а имеют полутона, перескакивают друг с друга, меняются от настроения, с возрастом, когнитивных установок и прочего. Соответственно это просто навешивание условных ярлыков, которые ещё и нестабильны.
Но в отличии мракобесия, типа карт таро, соционика имеет рациональное зерно. Поможет лучше понимать себя, других и взаимоотношения между людьми в социуме, если у тебя в этой части ноль знаний.
Аноним 16/09/22 Птн 20:08:44 #221 №565228 
>>565225
Ты себя заебешься типировать, будут тупо вероятности, которые меняются в зависимости от погоды на марсе.
Предсказаний поведения масс там и близко нет, максимум отношения 2-5 человек предположить.
Аноним 16/09/22 Птн 21:14:03 #222 №565230 
image.png
кiшка и мишка.mp4
На двух видео кошки ведут себя похоже, играя с мышкой переворачиваются на спину и смотрят ей в глаза.
https://youtu.be/X9VdlBXIS4I?t=74
Тут вообще нежно гладит лапой
Откуда такая игривость, зачем им вообще это? Смахивает на проявление эмоций и разума свойственных людям
Аноним 16/09/22 Птн 21:17:03 #223 №565232 
>>565230
Человек такое же животное как и они, разум есть у всех животных. Никакой принципиальной разницы между животными и человеком ни в чём нет. Ты религиозный верун или научпопер? Что тебя так удивило.
Аноним 16/09/22 Птн 21:18:48 #224 №565233 
>>565232
Хуя разрыв наукобота
Аноним 16/09/22 Птн 21:20:18 #225 №565234 
>>565230
> Тут вообще нежно гладит лапой. Откуда такая игривость, зачем им вообще это?

Не отделяй человека от животных, это вбитая в голову религией хуета, человек это обезьяна в пиджаке и не более, он так же гладит и любит корову, что бы съесть ее когда станет голоден.
Аноним 16/09/22 Птн 21:23:56 #226 №565235 
>>565234
Причем тут религия. Я спрашиваю зачем кошке такой паттерн, она же может упустить обед. Или это механизмы поведения сложились случайным образом в условиях изобилия и тепличных условий. Видел как львы так же играются с телятами и защищают их от сородичей
Аноним 16/09/22 Птн 21:28:31 #227 №565237 
>>565233
Ты чего порвалась дурочка тупая?
Аноним 16/09/22 Птн 21:29:04 #228 №565238 
>>565235
>. Я спрашиваю зачем кошке такой паттерн, она же может упустить обед.
Пиздоболка, ты не спрашивала это.
Аноним 16/09/22 Птн 21:29:58 #229 №565239 
>>565237
>>565238
Уроки делай, аутист малолетний
Аноним 16/09/22 Птн 21:32:20 #230 №565240 
>>565235
Она тупо не голодная. Ту же не ешь все что видишь пока еда не кончится?
Ещё у них инстинкт есть, приносить жертву поиграть детенышам, для обучения.
Аноним 16/09/22 Птн 21:37:37 #231 №565241 
>>565240
Добавлю.
Ещё в деревнях кошки приносят задушенную мышь хозяину. По бытовому считается, что показывает свою работу и нужность. По научному считает человека соплеменником и делится с ним едой, чтоб кожаный с голоду не сдох.
Аноним 16/09/22 Птн 21:52:44 #232 №565242 
изображение.png
>>564437 (OP)
Кстати, возможно ли будет стать азиатом в будущем с помощью crispr cas9 и гена EDER?
Аноним 16/09/22 Птн 22:28:23 #233 №565244 
>>565064
Бамп вопросу.
Аноним 16/09/22 Птн 22:59:02 #234 №565245 
>>565244
Тебе же сказали, что это ненаучный шизобред
Аноним 16/09/22 Птн 23:13:17 #235 №565247 
>>565244
Там написано "стремится к бесконечности", вполне научная используемая вещь, а не "бесконечное время".
Аноним 16/09/22 Птн 23:18:16 #236 №565248 
>>565242
Нужно изменить днк каждой клетки, для этого в процессе изменения нужно постоянно вливать новые порции криспра(или делать вирус саморазмножающимся, да так, чтобы не убивал клетки которые использует для размножения, а это опасно, мутация, и получаешь вирус-болезнь которая сама размножается), при этом нужно чтобы не было конфликта уже изменённых клеток и ещё не измененных, это наверное сложно, при смене рассовых признаков. Второе, нужно что крисп как-то отличал изменённые клетки от не изменённых, иначе он тупо будет опять и опять пытаться изменить уже изменённые, будешь вливать и вливать в себя кучи микстуры этой, а изменений после определенного этапа почти не будет, время полного изменения будет пиздец долгим. А чтобы отличал опять-таки, ещё сложнее наверное.
Аноним 17/09/22 Суб 04:43:43 #237 №565251 
>>565241
А зачем кошка срет в душевой если рядом лоток?
Аноним 17/09/22 Суб 09:05:35 #238 №565254 
>>565251
Обычная домашняя кошка в стандартных ситуациях так не делает. Кто-то где-то налажал.
Аноним 17/09/22 Суб 13:34:45 #239 №565255 
>>565248
Ох, как же круто...
Мимо биотехнолог, что мне учить читать?
Ибо учу по укр.. программе..
Аноним 17/09/22 Суб 14:58:50 #240 №565256 
>>565255
У меня очень поверхностные знания, ответил то что знаю, подробнее пусть другие если такие найдутся
Аноним 17/09/22 Суб 16:03:13 #241 №565257 
>>564437 (OP)
Тупейший вопрос эвер. Итак, допустим я встречаю высокоразвитых инопланетян или каких-то супермудрецов, которые все знают. Какие вопросы им задать, чтобы проверить, шизики это или реально сверхразвитые существа?
Желательно, из разных областей простые вопросы, на которые нет ответов, но наука эти вопросы уже поставила. И чтобы в будущем можно было проверить или уточнить что-то
Аноним 17/09/22 Суб 16:09:42 #242 №565258 
>>565257
https://ru.wikipedia.org/wiki/Задачи_тысячелетия

https://ru.wikipedia.org/wiki/Нерешённые_проблемы_современной_физики
Аноним 17/09/22 Суб 17:14:26 #243 №565261 
image.png
Вот смотрите, насколько я понял сегодняшний научный взгляд на зарождение жизни таков, что 4 с хуем миллиарда лет назад на Земле спонтанно возник саморепликатор, с которого всё и началось, и различные потомки которого начали эволюционировать, приспасабливаясь к окружающей среде, чтобы не сдохнуть?
Но я не могу понять, а почему какому-то куску материи, пускай у него даже и появилась возможность воссоздавать собственные копии, стало не похуй на собственное существование? Из чего у него вообще зародилось какое-то подобие того, что мы сегодня
зовём инстинкт самосохранения? Ведь без этого само понятие эволюции и жизни бессмысленно.
Аноним 17/09/22 Суб 17:37:27 #244 №565263 
>>565261
>Ведь без этого само понятие эволюции и жизни бессмысленно.
Что за бредятина? Инстинкт самосохранения нужен, чтобы успеть размножиться прежде, чем сдохнешь. Если организм статистически размножается быстрее, чем дохнет, никакое "самосохранение" не нужно. Как ты представляешь "самосохранение" при размножениии делением?
Аноним 17/09/22 Суб 17:38:48 #245 №565264 
>>565261
>спонтанно возник саморепликатор
Океан из компонентов репликаторов.
>Из чего у него вообще зародилось какое-то подобие того, что мы сегодня
>зовём инстинкт самосохранения?
У одних зародился, у других не зародился, угадай кто выжил.
На самом деле все, самосохранение есть только у животных.
Аноним 17/09/22 Суб 17:42:58 #246 №565265 
>>565261
>стало не похуй на собственное существование
Иначе сдохнешь.
Аноним 17/09/22 Суб 17:45:50 #247 №565266 
Как в однофазной сети меняются направления вращения в электродвигателях в стиралках? Что с щетками, что в безколлеторных.
Изменем допустим ноль и фазу местами, двигателю какая разница? Это ж одно и то же. Отдельно можно понять где ноль а где фаза, но когда провода соеденены, в сети переменного тока, нельзя понять где ноль а где фаза.
Аноним 17/09/22 Суб 17:48:59 #248 №565267 
>>565261

> Но я не могу понять, а почему какому-то куску материи, пускай у него даже и появилась возможность воссоздавать собственные копии, стало не похуй на собственное существование?

У репликатора-ДНК нет ни каких чувств. Чувства есть у его капсулы робота-носителя. Чувства появились эволюционными путём, так как дают громадное преимущество в выживаемости и распространении. Без чувства самосохранения долго не проживёшь и не распространишь ДНК своего хозяина, т.е. вымрешь.

Наверное ты вообще про квалиа и сознание. Тогда это «Трудная проблема сознания» и ответа на него нет, гугли сам.
Аноним 17/09/22 Суб 20:00:24 #249 №565268 
>>565267
>>565264
>>565263
То есть эволюция живых организмов в целом не обусловлена необходимостью их выживания?
Аноним 17/09/22 Суб 20:31:03 #250 №565269 
>>565268
Ты не понимаешь как всё работает. Нет никакой необходимости ни в чём и ничего, просто кто не выжил тот сдох.
Аноним 17/09/22 Суб 21:27:58 #251 №565271 
Испаряется ли сперма при кипячении или на раскаленной поверхности? Если кончить на асфальт под жарким солнцем, станет ли испарённая (если испаряется) сперма частью дождя? Проверить самостоятельно не могу, ибо я тян
Аноним 17/09/22 Суб 21:33:21 #252 №565272 
>>565271
Поразмыслив над 2 вопросом, я подумала, что, вероятно под солнцем испарится и станет дождем не сама сперма, а молекулы воды, содержащиеся в ней, а остальные частицы останутся на асфальте. Так ли это? И как насчёт харчка на асфальт, который пример участие в круговороте воды, в нём тоже будут только молекулы воды харчка, или это вообще всё работает по-другому и сперма и харчок вообще никак не испарятся?
Аноним 17/09/22 Суб 21:34:23 #253 №565273 
>>565272
*примет
быстрофикс
Аноним 17/09/22 Суб 21:38:13 #254 №565274 
>>565272
Да, именно так это и работает. Вода испаряется, вся органика остаётся на месте.
Аноним 17/09/22 Суб 21:41:26 #255 №565275 
>>565274
почти вся
Аноним 17/09/22 Суб 21:52:33 #256 №565276 
>>565274
Довольно странно кстати, по закону квадрата-куба молекулы должны быть очень тяжёлыми для испарения, и им должно быть намного тяжелее испариться, чем тому же свинцу поатомно содержащемуся в воде.
Но есть же легколетучие молекулы, тот же бензин, и ещё более летучие растворители.
Получается очистка воды дистилляцией полная хуйня, и там, допустим если была цель очистить воду от свинца, после дистилляции будет далеко нихера не почти чистая h2O, а до сих пор дохерище примеси свинца.
Аноним 17/09/22 Суб 23:29:38 #257 №565280 
16620740919111.jpg
Фугасное и бризантное взрывное действие.
Первое как я понял отталкивающее, второе-дробящое на части.
Почему для балона с газом нет дробящего действия.?
Какие ВВ разрывают тело на части?
Аноним 18/09/22 Вск 00:46:48 #258 №565285 
>>565280
Фугасность это сила ударной волны или плотность энергии, переходящая в взрыв.
Бризантность это качество ударной волны - скорость/ширина границы разрыва/перепад давления.
Бризантные взрывчатки способны создавать очень быструю ударную волну(в десятки раз превышает скорость звука) и передать ее объекту. За счет своей сверхскорости ударная волна порождает качество другие разрушения.
Скорость ударной волны в газах ограничена скоростью звука в газах. (Есть исключения!)
На счет разрыва тушки, тут важен больше градиент и энергия взрыва. Обычный пороховой заряд на ура отрывает пальцы и кисти.
Аноним 18/09/22 Вск 01:23:43 #259 №565290 
16620740919111.jpg
>>565285
Но почему порох отрывает, а газ нет?
Где-то уже слышал такое.
Аноним 18/09/22 Вск 01:28:15 #260 №565292 
16620740919111.jpg
>>565285
>Скорость ударной волны в газах ограничена скоростью звука в газах. (Есть исключения!)

Ты хочешь сказать что ударная волна взрыва газа такая же как и скорость звука? Я думал что сгорание газа любого обьема моментальное. Следовательно и энергия взрыва максимально близкая к энергии скорости сгорания тротила или пороха.
Аноним 18/09/22 Вск 01:28:39 #261 №565293 
>>565290
Скорость волны детонации. Газ мягко сгорает, медленно, он скорее горит, нежели взрывается, в общепринятом смысле, хотя разницы между взрывом и горением на самом деле нет.
Аноним 18/09/22 Вск 01:35:56 #262 №565294 
>>565292
>моментальное
Ага, быстрее скорости света в вакууме.
Нету такого в природе, моментального. Не существует.

>Ты хочешь сказать что ударная волна взрыва газа такая же как и скорость звука?
Он не прав, он там мельком написал, но то основной фактор а не исключение. Скорость ударной волны в случае твердых и жидких взрывчаток достаточно близко возле них намного выше чем скорость ударной волны которая потом в воздухе идёт, за счёт скорости детонации в малом объёме. А газ не может быстро сдетонировать, он горит со скоростью звука в воздухе примерно, это в принципе мало, так ещё и не везде сразу,, а постепенно во всём объеме, а объём большой, значит плотность энергии маленькая в каждой точке.
Аноним 18/09/22 Вск 02:08:18 #263 №565295 
16620740919111.jpg
>>565294
>А газ не может быстро сдетонировать, он горит со скоростью звука в воздухе примерно
Пиздец, аналогичное мышление дало трещину.
Аноним 18/09/22 Вск 11:25:52 #264 №565302 
Сап двощ. Учусь на инженера и на лекциях по физике постоянно говорят брать учебники только в системе СИ. Вопрос - а зачем в принципе нужна СГС, неужели удобнее?
Аноним 18/09/22 Вск 18:51:30 #265 №565308 
>>565302
>неужели удобнее?
да
Аноним 18/09/22 Вск 20:21:02 #266 №565313 
Как найти детальное описание конструкций башен-близнецов? Планы, разрезы, описание материалов, хочу в программном комплексе создать модель и попробовать испытать на особые воздействия, благо железо позволяет.
Не знаю на какую доску лучше написать, поэтому напишу наиболее близкую по смыслу.
Аноним 18/09/22 Вск 20:56:22 #267 №565316 
>>565313
Конспирологией увлекаешься?
Аноним 18/09/22 Вск 21:04:51 #268 №565317 
>>565313
За 20+ лет наверняка кто-то уже это делал, проверь перед тем как заниматься самому.
Аноним 19/09/22 Пнд 04:38:32 #269 №565330 
башни 11 сентября.webm
сентябрь.mp4
>>565313
Я тебе сразу могу сказать как они сложились. Пожар размягчил металлический каркас и задание получило критическую нагрузку. Каркас заделан на саморазрушение в случае пиздеца. Бетонированная часть тоже. Там заклепки вылетают и верхний этаж ломает нижний. Думаю зачем это сделано объяснять не нужно
Аноним 19/09/22 Пнд 04:45:50 #270 №565331 
image.png
image.png
>>565317
Диаграмма NIST, показывающая изгиб колонны 79 (обведено оранжевым), в результате которого началось прогрессирующее разрушение здания
https://ru.wikipedia.org/wiki/%D0%A0%D0%B0%D0%B7%D1%80%D1%83%D1%88%D0%B5%D0%BD%D0%B8%D0%B5_%D0%B1%D0%B0%D1%88%D0%B5%D0%BD_%D0%92%D1%81%D0%B5%D0%BC%D0%B8%D1%80%D0%BD%D0%BE%D0%B3%D0%BE_%D1%82%D0%BE%D1%80%D0%B3%D0%BE%D0%B2%D0%BE%D0%B3%D0%BE_%D1%86%D0%B5%D0%BD%D1%82%D1%80%D0%B0_%D0%B2_%D0%9D%D1%8C%D1%8E-%D0%99%D0%BE%D1%80%D0%BA%D0%B5
Аноним 19/09/22 Пнд 05:19:30 #271 №565332 
>>565330
Сила тяжести g, здание стоит. Значит его прочность больше g, плюс запас прочности. Когда верхние этажи начинают падать, то падают они с силой g, а сопротивляется их падению конструкция прочностью g+.
Да, оно будет проламывать, т.к. разгон набрало и сила теперь чуть больше, но их должно нихуёво тормозить во время падения постоянно, та же сила g, да ещё запас прочности.
А том насколько знаю основная проблема в том что этажи падали буквально с ускорением свободного падения. Будто конструкции никакой и не было, или будто она постоянно взрывалась чтобы не мешать падению.
Аноним 19/09/22 Пнд 05:52:18 #272 №565333 
>>565332
Ты себе примерно представь вес этой башенки. Конечно она будет складываться как карточный домик. Тут только можно похлопать инженерам, иначе бы домик завалился на бок и по принципу домино уронил бы еще много домиков
Аноним 19/09/22 Пнд 06:26:57 #273 №565334 
>>565333
Ты даже не понимаешь о чём речь.
Аноним 19/09/22 Пнд 06:34:14 #274 №565335 
NISTWTC7collapsemodelwithdebrisimpactdamage.ogv.240p.vp9.webm
>>565334
Я понимаю что тебе что-то показалось. Выше ссылка на обширную статью, где эту башню по винтикам разбирают. Тебе конечно лень читать, проще поверить в рязанский сахар
Аноним 19/09/22 Пнд 06:39:57 #275 №565336 
Почему всем животным можно иметь подвиды, а человеку нельзя. Как только ученый говорит о расах, его сразу обвиняют в расизме.
Дарвин расист.
Чел который открыл днк расист.
Мерить черепа нельзя, мы же не фашисты.
Гнобить белых можно, они же белые хуемрази
Мерять интеллект нельзя. Ну даже если померил, нельзя проводить статистику, вдруг у голожопых родственников Обамы айкью ниже.
Торжество науки? Нет, торжество коричневых новиопов и полукровок. Ученых-колдунов на костер
Аноним 19/09/22 Пнд 07:07:44 #276 №565337 
>>565336
> у голожопых родственников Обамы айкью ниже.
Ниже, чем у тебя? Вот это вряд ли.
Аноним 19/09/22 Пнд 07:44:59 #277 №565338 
>>565337
чурка обиделась что ее родню задели
Аноним 19/09/22 Пнд 10:56:27 #278 №565342 
>>565338
Лучше, когда Обама в родне, чем двачер-нацик.
Аноним 19/09/22 Пнд 11:10:00 #279 №565346 
>>565337
>Ниже, чем у тебя?
Объясни почему за 200 000 лет истории Африки там не построили никакой цивилизации? А те города, что возникли, исчезли навсегда и о них знаю полторы калеки археолога?
мимо
Аноним 19/09/22 Пнд 15:52:23 #280 №565350 
>>565346
Жарко там. Зашли туда макаки и вышли со словами, "Вот это парилка нахуй"
Аноним 19/09/22 Пнд 17:16:41 #281 №565355 
>>565335
Ты не понимаешь смысла слов а советуешь статью, лол.
Аноним 19/09/22 Пнд 17:17:25 #282 №565356 
>>565342
Так ты просто куколд-хуесос, лол
Аноним 19/09/22 Пнд 18:57:18 #283 №565357 
>>565355
лолкуящая малолетняя дебилка, зайди в зогач и там спроси, кек
Аноним 19/09/22 Пнд 19:18:28 #284 №565358 
>>565332
У тебя очень поверхностное представление о механике зданий. Оно в строительстве не рассматривается как один сплошной объект с прочностью "g". Помимо прочности на сжатие колонн, есть продавливание, изгибы, кручение, устойчивость, опрокидывание итд. И разбивается здание даже если не на конечные элементы, а на стержни и пластины, которые работают как единая связевая система. Ко всему этому прикладываются нагрузки, от них возникают напряжения. Если напряжение превышает предел прочности, то происходит разрушение.
Помимо прочности, есть ещё одна группа предельных состояний - по деформациям. То есть коллапс сооружения может произойти не из-за разрушения плит и балок, а из-за крена, изгиба колонн, которые тянут за собой остальную конструкцию и ослабить её в другом месте.
Я конечно сам ещё не такой классный строитель, но смысл в моделях обрушения вроде вижу. >>565335
На гифке видно как выгибается колонна под воздействием нескольких рухнувших перекрытий и запускает весь каскад.
Аноним 19/09/22 Пнд 19:18:42 #285 №565359 
>>565357
Тупая дурочка, не рвись.
Аноним 19/09/22 Пнд 19:51:08 #286 №565360 
>>565358
>Оно в строительстве не рассматривается как один сплошной объект с прочностью "g".
Ты хочешь спорить с тем что здание прочнее чем g?
Аноним 19/09/22 Пнд 20:19:13 #287 №565362 
>>565359
>ти по рвался
я говорю, зайди в zog и спроси. только не лолкай, а то поймут что ты школота и лапши на вешают
Аноним 19/09/22 Пнд 20:21:30 #288 №565363 
>>565362
Дурочка тупая, не рвись.
Аноним 20/09/22 Втр 00:06:05 #289 №565369 
20220920000426.jpg
Если человек произошел от обезъяны, то куда делись промежутки и почеиу более примитивные обезьяны выжили, а промежутки нет?
Аноним 20/09/22 Втр 00:29:47 #290 №565371 
>>565369
То ж разные виды. Сейчас живут не те из которых человек получился. И они не примитивные, потому что человек не развитый, он просто другой.
Средних нету потому что не может конкурировать ни с более обезъяннего вида, ни с человеком.
Человек раньше мог быть "средним", потому что людей не было. И в чём-то он у обезьян отсасывал, собственно поэтому они и не вымерли.
Аноним 20/09/22 Втр 00:31:26 #291 №565372 
>>565369
>то куда делись промежутки
Они стали человеком.
Или вымерли, если были другие ответвления кроме человека на каком-то этапе.
Возможно есть ещё те кто на каком-то этапе от более человечного обратно в более обезьянную скатился сторону.
Аноним 20/09/22 Втр 00:34:48 #292 №565373 
054f6b56e83d05d83a44eb07aaefc76e-1.jpg
Какой фенотип у Алексея Столярова?
https://ru-sled.ru/fenotipy-evropeoidnoj-rasy-evropy/
Аноним 20/09/22 Втр 00:42:41 #293 №565374 
>>565371
А предпоследние не могли заниматься охотой и тем самчм сохранить свой вид? То есть по сути как мне кажется это был основной критерий выживаемости вида.
Аноним 20/09/22 Втр 00:57:51 #294 №565375 
>>565374
Они не могли выжить потому что человек уже тогда был.
Это если кто-то паралельно вместе с человеком стал почти таким как он, на ступень ниже.
И с обезьянами не мог конкурировать. Человек мог на таком же этапе, потому что тогда не было человека следующего этапа.
Охотиться могли, но вопрос не в самом факте возможности охотиться, а в конкуренции и в эффективности.
Им только или обратно обезьянами становиться или вымирать.
Аноним 20/09/22 Втр 02:21:32 #295 №565378 
Все неорганические вещества с их молекулярной структурой можно просчитать как какие-нибудь белки той же роззетой? Могли бы получить свойства всех существующих и несуществующих материалов.
Аноним 20/09/22 Втр 07:00:21 #296 №565381 
>>565369
>Если человек произошел от обезЬяны
От ардипитека.
>куда делись промежутки
Человек уничтожал всех конкурентов претендующих на занимаемую нишу.
>более примитивные обезьяны выжили
Потому что не являлись прямыми конкурентами.
Аноним 20/09/22 Втр 07:33:10 #297 №565382 
Можете объяснить линейность коэффициента трения k вне зависимости от нагрузки? Коэффициент трения скольжения стали по стали будет 0.17 внезависимости от того, давите вы друг на друга с силой 1 Н, 1 кН, 1 МН или 10 МН. Почему?
Аноним 20/09/22 Втр 09:03:40 #298 №565383 
>>565381
Почему обезьяны не прямые конкуренты, они поедают раков, ягоды и фрукты, которые мог бы сьесть человек? При этом сьемть обезьяну нельзя, в дикой природе они переносят болезни.
Аноним 20/09/22 Втр 09:29:49 #299 №565386 
>>565382
А чем тебя стандартное объяснение через кулоновское отталкивание не устраивает?
Поверхности находятся в атомарно тесном контакте только на небольшой части их общей площади, и эта площадь контакта пропорциональна нормальной силе (до насыщения, когда вся площадь будет находиться в атомном контакте).
Аноним 20/09/22 Втр 09:44:27 #300 №565387 
>>565383
>Почему обезьяны не прямые конкуренты
Не изготовляют орудий труда и не применяют оружие. Это главное отличие человека от его предков - использование орудий. Биологические различия неочевидны
Аноним 20/09/22 Втр 09:55:17 #301 №565389 
>>565387
>Не изготовляют орудий труда и не применяют оружие.
Изготавливают и применяют.
Аноним 20/09/22 Втр 10:12:42 #302 №565390 
>>565389
Применяют, но не изготавливают. Максимум палку/ветку могут сломать.
После использования сразу выкидывают.
Топология элементарных частиц Аноним 20/09/22 Втр 12:13:55 #303 №565393 
атом.png
Сап, двач. Может ли кто-нибудь запилить структурную схему иерархии элементарных частиц открытую на текущий момент типа пикрелейтеда. Гуглятся только какие-то таблицы на которых ничего не понятно.
Аноним 20/09/22 Втр 13:36:44 #304 №565395 
>>565393
Там ещё только 2 кварка, и всё, в разной комбинации по 3 штуки в нейтроне и протоне, udd и uud, электрон фундаментален.
Ещё есть 20 где-то видов кварков, но они нигде не используются в материи. Откуда они "вылетают" при экспериментах я хз.
Все остальные частицы условно вылетают из взаимодействий нейтронов и протонов, или т.к. они сомтоят из кварков, то из взаимодействий еварков, но их там нету внутри протонов и нейтронов "в составе", они именно что вылетают из-за взаимодействия. Появляются.
Аноним 20/09/22 Втр 13:42:06 #305 №565397 
>>565390
Изготавливают, они не изготавливают орудие для изготовления орудия которое потом уже используют для конкретного дела типа добычи еды.
Уровень абстракции максимум 1. 2го уровня нету кроме как у людей.
Аноним 20/09/22 Втр 13:49:34 #306 №565398 
атом.png
>>565395
а ты можешь исправить схему если она с ошибками + написать на схеме названия частиц ,которые сами по себе вылетают неизвестно откуда, типо нейтрино, для полноты картины
Аноним 20/09/22 Втр 14:01:11 #307 №565399 
>>565398
Там не 3 uud
uud и ddu это уже 3 кварка, u-кварк и d-кварк, в одной клетке у тебя должна быть или d или u, итого 3.

Про другие частицы посмотри в ютубе видео Козакова про стандартную модель.
Аноним 20/09/22 Втр 14:13:45 #308 №565400 
>>565399
А как же глюоны?
Масса нуклонов в первую очередь это потенциальная яма сильного взаимодействия.
Аноним 20/09/22 Втр 14:24:12 #309 №565401 
>>565400
Они ж вроде не существуют в свободном виде? Как и кварки внутри протонов и нейтронов довольно условно выделили как "составляющие части" протона и нейтрона. Вроде они там при каких-то экспериментах просвечиваются, что именно 3 шт чего-то внутри, а вроде и нет, не помню, это мне надо вспоминать, смотри Казакова видео про стандартную модель.
Аноним 20/09/22 Втр 16:54:27 #310 №565407 
>>565401
Если нуклон сталкивать с ультрарелятивистскими частицами, то он будет порождать так называемые адроные струи - в основном всякие мезоны.
Когда запилили очень чувствительные детекторы и набрали статистику, то выяснили, что мезоны в адронных струях появляются не сразу, а через которое время после столкновения. Также замечали странности с самим нуклоном, во время столкновения он распадался на несколько независимых частей + нечто, из чего потом рождается всякая не адронная хрень(пары частиц).
Вот это нечто и есть комбинации глюонов в разных конфигурациях.
Когда стали сталкивать на БАКе уже кучи протонов и антипротонрв, то по статистики смогли зафиксировать прямо взаимодействие глюонов с другим адроном до того, как они стали мезонами или распались на что-то другое.
В некоторое смысле глюоны в свободном виде задетектины прямо в отличие от кварков.
Аноним 20/09/22 Втр 17:06:50 #311 №565408 
чурки в естественной среде.mp4
>>565390
ни разу не видел чтоб обезьяна дралась палкой
Аноним 20/09/22 Втр 21:40:24 #312 №565414 
1.jpg
>>565408
Ну и помогла этой обезьяне-омежке та палка?
Ерохинян бабизяноидзе просто отхуячил на 1.34.
Аноним 20/09/22 Втр 21:55:44 #313 №565415 
>>565378
Розетта заточена под белки, если ты в принципе о возможности посчитать говоришь, то да, теоретически имея бесконечно-мощный комп, ты просто в него уравнение шредингера запихиваешь для атомов в молекуле и все свойства получаешь на бумажечке какие только захочешь узнать.

В реальности же, там так быстро растет сложность расчетов, что для большого количества атомов надо брать упрощенные уравнения, которые уже неточные.
Аноним 21/09/22 Срд 15:00:50 #314 №565435 
image.png
>>565407
>нуклон
>мезон
>адрон
>глюон

это всё фундаментальные частицы возникающие в ходе экспериментов, не имеющие иерархии как в стандартной модели?
Аноним 21/09/22 Срд 15:03:46 #315 №565436 
>>565414
>Ерохинян бабизяноидзе
Не понял.
Но факт, что пиздиться палкой может додуматься.
Аноним 21/09/22 Срд 15:12:40 #316 №565437 
StandardModelofElementaryParticlesru.svg.png
>>565435
Фундаментальные частицы только на пикче. В твоем списке только глюон. Косвенно он регистрируется в экспериментах.
Нуклон это протон или нейтрон, состоят из трех кварков и "глюонного мешка".
Мезоны это всякие пионы и далее, состоят из двух кварков.
Адроны это любые составные частицы, участвующие в сильном взаимодействие. Сюда включается нуклоны, мезоны, всякие гипероны и прочее говно. Также сюда относятся гипотетические частицы вроде экзотических мезонов.
Все частицы включаются в Стандартную модель.
Аноним 21/09/22 Срд 15:33:04 #317 №565438 
>>565436
А человек без культуры может и не додуматься.
Аноним 21/09/22 Срд 15:34:13 #318 №565439 
>>565437
> электрон фундаментален.
>Фундаментальные частицы только на пикче.
>Все частицы включаются в Стандартную модель.
очень интересно но не понятно какую иерархию эти частицы образуют.
Аноним 21/09/22 Срд 18:01:50 #319 №565440 
>>565439
>не понятно какую иерархию эти частицы образуют.
Кварки и лептоны делятся на три поколения. Остальная систематика фундаментальных частиц не иерархична.
Аноним 21/09/22 Срд 18:18:02 #320 №565441 
image.png
>>565440
>и лептоны

куда на схеме встраивается лептон?
Аноним 21/09/22 Срд 19:29:24 #321 №565445 
изображение2022-09-21192908531.png
>>565441
Ну мне не сложно картинку запостить ещё раз.
Аноним 21/09/22 Срд 19:41:15 #322 №565447 
>>565445
Их же больше намного
Аноним 22/09/22 Чтв 10:58:37 #323 №565466 
>>565447
Это все фундаментальные частицы Стандартной модели. Их не больше и не меньше. Мезоны и барионы не являются фундаментальными.
Гипотетических частиц за пределами Стандартной модели понапридумывано много, но существование ни одной из них не подтверждено.
Аноним 22/09/22 Чтв 13:08:52 #324 №565476 
>>565445
Ну мне не сложно картинку запостить ещё раз.

Она не полная, на этой картинке нет нейтрона, протона и электрона. Добавь их туда, пожалуйста
Аноним 22/09/22 Чтв 14:56:11 #325 №565477 
>>565476
>Она не полная, на этой картинке нет нейтрона, протона и электрона. Добавь их туда, пожалуйста
Я, конечно, понимаю, что это тред для тупых вопросов, но ты переигрываешь. Тебе раз за разом говорят, что там не может быть протона и нейтрона (электрон там есть), а также не может быть атома серы, бутылки водки, слона или галактики.
Аноним 22/09/22 Чтв 16:26:20 #326 №565481 
image.png
>>565477
Где тут видишь здесь электрон? В пеинт можешь?
Аноним 22/09/22 Чтв 17:19:17 #327 №565484 
>>565481
Первый столбец третья строка.
Аноним 22/09/22 Чтв 18:32:06 #328 №565486 
Сап. Есть проблема, мне в чулане понадобилась розетка, а ее там нет. Но есть лампа. Недолго думая, я ее демонтировал, а к проводам из стены приделал корпус розетки. При подключении к ней, кнопка не сетевом фильтре горит, но тусклее, чем при подключении к другим розеткам. Когда подключаю какой либо потребитель, кнопка гаснет и ничего не происходит. Напрямую в розетку потребители тоже не работают.
Неужели в этих проводах течет именно свет, а не электричество?
Аноним 22/09/22 Чтв 20:03:20 #329 №565489 
>>565486
Ты может последовательно подключил а не паралельно
При последовательном подключении напряжение падает в 2 раза, при параллельном ток увеличивается в 2 раза.
Аноним 22/09/22 Чтв 20:07:40 #330 №565490 
изображение.png
>>565408
>ни разу не видел чтоб обезьяна дралась палкой
Аноним 22/09/22 Чтв 20:14:13 #331 №565491 
>>565489
Там вариантов особо нет, из стены торчит двужильный провод, так что ума ни приложу, что я сделал не так
Аноним 22/09/22 Чтв 21:32:26 #332 №565494 
1.jpg
>>565436
>Не понял.
на 1 минуте 34 секунде обезьяны без палки, вломила обезьяне с палкой. Причем так, что обезьяна с палкой ту палку потеряла и сбежала. Вывод: похуй додумалась обезьяна до палки или нет. Пизды вломит тот у кого силы больше. Сила в силе! а не в мозгах
Аноним 22/09/22 Чтв 22:00:41 #333 №565495 
изображение.png
>>565494
Блицаю всех с помощью серной кислоты, которая в балоннах из под перцовок.
Доблициваю с помощью телескопа с наточенным наконечником.
Аноним 22/09/22 Чтв 22:13:02 #334 №565496 
>>565490
ну замахнулся, а где удар
Аноним 22/09/22 Чтв 22:13:23 #335 №565497 
слонопотам.mp4
>>565490
Аноним 22/09/22 Чтв 23:27:04 #336 №565500 
700NitroExpress-4.jpg
>>565494
>Пизды вломит тот у кого силы больше.
Аноним 23/09/22 Птн 05:58:32 #337 №565507 
Screenshot47.png
Почему так?
Аноним 23/09/22 Птн 06:26:54 #338 №565509 
>>565507
Камни хранятся дольше космических кораблей.
Аноним 23/09/22 Птн 06:34:23 #339 №565510 
>>565509
Конспиролог, плес
Аноним 23/09/22 Птн 08:40:27 #340 №565511 
>>565510
Ну технически он прав.
Допустим есть два варианта - первый, это когда КК приземляется, все кто в нем были мертвы и он просто остается стоять на месте.
С ним реально может произойти что угодно, вплоть до того, что он приземлился в болоте и его засосало и он там покоится где-нибудь тысячелетия. Но найти его не представляется возможным.
Второй вариант, когда КК упал на большой скорости.
Скорее всего последует если не взрыв, то точно практически полная деформация корабля а уж если скорость 2-я космическая, то его размажет тонким слоем по поверхности.
За 50 000 лет вода с ветрами этот тонкий слой полностью уничтожат.
Аноним 23/09/22 Птн 08:48:12 #341 №565514 
>>565511
хуйню не мели. вопрос был почему на способы обточки камней ушло времени больше, чем вылезти из каменного века в космос.
то что камень дольше огромного куска железа сохранится это бабушке своей расскажешь
Аноним 23/09/22 Птн 11:59:00 #342 №565520 
>>565514
>то что камень дольше огромного куска железа сохранится это бабушке своей расскажешь
Чувак, камень, это алюмосиликат с вкраплениями.
Он инертен нахуй.
А кусок железа это оксид железа, который даже если ты не будешь красить каждый год в простой морской воде за 10 лет растает.
По поводу ответа на твой вопрос - тут надо понимать, что он напрямую зависел от скорости эволюции мозга.
Аноним 23/09/22 Птн 12:02:03 #343 №565521 
>>565520
Да это вообще не ответ. Лучшебы не разевал свою пасть, чем такую хуйню высирать
Аноним 23/09/22 Птн 14:40:33 #344 №565524 
>>565521
Ты просто слишком тупой, чтобы понять, сорян.
Аноним 23/09/22 Птн 15:23:09 #345 №565525 
>>565507
Развитие есть процесс нелинейный. А, как вариант, экспоненциальный. Пока нихуя нет все развивается пиздец медленно, а как что-то появилось, так сразу в космос полетели.
Аноним 23/09/22 Птн 18:54:08 #346 №565529 
>>565524
Зато я ебу твою мать
>>565525
А подробнее можно?
Аноним 23/09/22 Птн 18:59:29 #347 №565530 
>>565529
Что подробнее? На основе приведенных тобою экспериментальных данных я могу придумать только такую гипотезу, их объясняющую. Для ее уточнения или опровержения необходимо больше данных. Либо обратиться к более умному человеку.
Аноним 23/09/22 Птн 19:59:02 #348 №565532 
>>565530
Это не экспериментальные, а палеонтологические. Тут достаточно среднего человека для объяснения, просто мне лень думать, хочется чтоб кто-нибудь за меня
Аноним 23/09/22 Птн 21:09:42 #349 №565535 
>>565514
>почему на способы обточки камней ушло времени больше
Потому что ни один нормальный человек не будет заниматься такой хуитои больше необходимого.
Аноним 24/09/22 Суб 09:12:23 #350 №565546 
bandicam 2022-09-24 08-59-52-813.mp4
65tyh.png
Что значит "захват"? Протон поглощает электрон? Получается электрон таки падает на ядро?
Аноним 24/09/22 Суб 09:50:21 #351 №565547 
>>565546
>Что значит "захват"?
Взаимодействие.
>Протон поглощает электрон?
p + e → n + νe
>Получается электрон таки падает на ядро?
Не получается. Электрон всегда находится в ядре с некоторой вероятностью. Если данная реакция энергетически выгодна, то она будет происходить (тоже с некоторой вероятностью).
Аноним 24/09/22 Суб 10:15:36 #352 №565548 
>>565546
Это условное название, обобщенно идет взаимодействие u-кварка из протона и электрона с превращением его в d-кварк и нейтрино. Это происходит через слабое взаимодействие, у которого есть некоторый порог по энергии. И реакция энергически не выгодная.
Если электрон и протон столкнуть на больших энергиях, то возможно превращение его в нейтрон и нейтрино, но вероятность такой реакция мала из-за того же порога.
В атоме электрон уже "упавший" и даже больше, у него отрицательная энергия, что порождает его дискретный спектр тем самым блокирует многие его взаимодействия. Чтоб электрон с протоном взаимодействовал нужно электрона была положительная энергия, ну или чтоб дискретные уровни были достаточно широкими.
Аноним 24/09/22 Суб 15:39:40 #353 №565552 
>>565507
Там и так наблюдается прогрессия если ты не заметил. В результате закономерный итог. Ничего удивительного.
Аноним 24/09/22 Суб 16:10:03 #354 №565553 
супертанк.webm
>>565552
Что нас ждет лет через 500 примерно можно представить?
Аноним 24/09/22 Суб 16:26:49 #355 №565554 
>>565553
Ничего особенного, законы вселенной не бесконечны чтобы постоянно что-то новое открывать, и возможности не бесконечны которые дают владение законами физики.
Мы не наблюдаем нигде во вселенной суперкомпактной доступной энергии, не наблюдаем супер быстрых перемещений и такого плана. Изучили животных и природу, изучили космос, повторили в слегка изменённом виде с пользой для себя.
Всё. Ты в космосе видишь какие-то чудеса? Ну чёрные дыры, толку нет. Ну звёзды-очень слабый источник энергии относительно размеров. Никаких чудес. Если это не образовалось само, значит это невозможно.
Раньше просто и этого не знали, ничего почти, просто как бы не обращали внимание на то что вокруг, теперь обратили внимание, и всё на что можно было обратить внимание закончилось, знаем практически всё.
Аноним 24/09/22 Суб 16:31:11 #356 №565555 
>>565553
Танк как на видео кстати невозможен по закону квадрата-куба.
Аноним 24/09/22 Суб 16:33:21 #357 №565556 
>>565554
> знаем практически всё.
Проиграл. Чел тупо истории не знает
Аноним 24/09/22 Суб 16:33:52 #358 №565557 
>>565555
Турбины на танки давно приделали
Аноним 24/09/22 Суб 16:45:41 #359 №565558 
>>565556
Ты даун, рассуждения сколько ангелов на кончике иглы и знать фундаментальные константы вселенной это разные вещи, и позволяет говорить что "сейчас знаем практически всё".

Есть квантовая теория, есть теория относительности. То что их не получается объединить и что невнятый переход между ними, не значит что появится что-то ВАУ. Они вместе всё объясняют. Ну будет одна теория вместо двух всё объяснять. Что изменится?
Аноним 24/09/22 Суб 16:46:12 #360 №565559 
>>565557
Ты тупой, сорян. Речь не о турбинах.
Аноним 25/09/22 Вск 14:25:51 #361 №565581 
>>565554
>законы вселенной не бесконечны
Бесконечны, т.к. не может существовать теории всего. Теорема Гёделя о неполноте как бы говорит нам, что с любыми законами (записанными в виде математики) мы сможем мысленно или даже реально создать систему физическую, для которой законы записать можно, но решить или доказать что решение есть (т.е. предсказать поведение) будет невозможно.
>>565558
В 1900 году считалось, что физика открыла все законы уже, и осталась небольшая проблема с излучением, но если её поправить, то физика "Завершится".
Аноним 25/09/22 Вск 18:24:39 #362 №565584 
>>565581
> В 1900 году считалось, что физика открыла все законы уже

Ну как бы так оно и есть, пик последней научной революции тогда и был, сейчас мы на этапе ее стагнации, технического освоения сделанных открытий и их уточнений. Ни каких фундаментальных открытий последние 100 лет не было.
Аноним 25/09/22 Вск 20:26:56 #363 №565585 
>>565581
>Бесконечны, т.к. не может
Бесконечны возможно приближения и интерпретации уточняющие, но точно не законы по которым работает вселенная. Ты похоже даже не видишь в чём разница.
Аноним 25/09/22 Вск 21:10:45 #364 №565587 
>>565558
>>565584
какой упоротый даун
Аноним 26/09/22 Пнд 03:52:16 #365 №565592 
>>564437 (OP)
Как в разы увеличить время возможного нахождения в комнате с высоким угарным газом, если нет акваланга, или типа того? Хочу затестить детектор.
Аноним 26/09/22 Пнд 11:27:20 #366 №565594 
>>565592
Любой пожарный противогаз должен фильтровать CO.
Аноним 27/09/22 Втр 02:03:39 #367 №565613 
>>565587
Ты чего порвалась дурочка тупая?
Давай по пунктам аргументы, иначе тупая шалава.
Аноним 27/09/22 Втр 07:20:17 #368 №565618 
>>565613
>аргументировать для малолетнего школопёздика
no
Аноним 27/09/22 Втр 10:12:17 #369 №565622 
>>564437 (OP)
Реквестирую тему для курсовой работы по зоологии.
Аноним 27/09/22 Втр 13:13:28 #370 №565624 
Сап, почему арены жидкие или твёрдые?
Почему бензол вообще может быть в жидкой фазе?
Аноним 27/09/22 Втр 13:43:51 #371 №565625 
>>565618
Молодец дурочка тупая, доказала что ты тупая шалава.
Аноним 27/09/22 Втр 22:04:12 #372 №565641 
Человек по агрегатному состоянию кто? Для твердого тела у нас вроде нет решетки, для аформного мы не растекаемся, жидкости в нас дохуя, но форму сосуда мы принимаем лишь ограниченно (ну типа если надо, то можно поизгибаться, чтоб сильнее к стенкам прилипнуть) и можем вообще ее не принимать.
Аноним 27/09/22 Втр 22:06:02 #373 №565642 
>>565641
>нет решетки
есть
Аноним 27/09/22 Втр 22:09:19 #374 №565644 
>>565642
Рили? Какая она?
Аноним 27/09/22 Втр 22:22:39 #375 №565647 
>>565644
Многие биополимеры, в частности белки образует сорт оф молекулярные кристаллы. На ОП-вебмке >>564437 (OP) белок ползет по микротрубочке, которая состоит из двух простых белков, которые могут самопроизвольно соединяются с другом в линейные структуры. Из микротрубочек делается внутренний клеточный скелет.
Другой пример это билипидный слой мембран, это по сути молекулярный 2-д кристалл.
Или целлюлоза у растений или коллаген у животных.
Аноним 28/09/22 Срд 05:05:44 #376 №565664 
A30BJJq.png
Сколько стоит литр тестостерона приемлемой для употребления чистоты? Дохуя ведь? А взгляните, какая простая молекула, это вам не белок ботулотоксина, не космический шатл, а просто три элемента в незатейливой конфигурации. Так а хуле ваша мегапродвинутая наука не может сделать такое простое вещество за нидораха? В чём основные подводные камни химии (особенно органической), кароч? Почему школьная химия напрочь не работает?
inb4 дрищ спок
Аноним 28/09/22 Срд 08:46:11 #377 №565667 
>>565553
Чё за фильм?
Аноним 28/09/22 Срд 08:50:30 #378 №565668 
>>565664
Дрищ, спок.
Что значит сделать за нидорага?
Во сколько ты оцениваешь синтез килограмма?
Как оценивал? Трудились у тебя при синтезе рабы или нобелевские лауреаты, которым будь добр плати.
Аноним 28/09/22 Срд 08:53:21 #379 №565669 
>>565584
>Ни каких фундаментальных открытий последние 100 лет не было.
Перечисли критерии фундаментальности.
мимо
Аноним 28/09/22 Срд 09:02:53 #380 №565670 
>>565667
Унесенные ветром
Аноним 28/09/22 Срд 09:24:00 #381 №565672 
>>565668
В цену начальных элементов (полторы копейки)*хз какой-то параметр сложности молекулы, по картинке она весьма простая. Ну т.е. как троечник рассуждаю, что если по одному атому присоединять, её же просто сделать, это ж не белок какой ебучий. По аналогии с 3д принтерами, послойно делающими простые механизмы,- вот и эта молекула простая, почему её нельзя?..
Ну и вот почему такая логика completely булшыт? Вплоть до того, что тесто это до сих пор с яиц быков доят вроде, а не сами делают?
у меня к тому же шизоаналогия, что химия в этом с политотой схожа, когда диванному кажется, что надо просто взять и собрать кучку умных людей в идеально слаженный коллектив, который всех накажет и всё порешает, и просто не мешать им, ведь это же так просто,- но почему же не работает? Та не иначе рептилоиды руинят
Аноним 28/09/22 Срд 09:26:50 #382 №565673 
>>565672
>по картинке она весьма простая
>как троечник рассуждаю
>у меня к тому же шизоаналогия
Это многое объясняет.
>>565672
>просто
Если это так просто, то почему ты этого еще не сделал?
Аноним 28/09/22 Срд 09:31:07 #383 №565674 
>>565672
>Ну и вот почему такая логика completely булшыт?
Поясняю по хардору.
У тебя логика уровня - почему нельзя взять и без задней мысли создать антиматерию и просто и без задней мысли её аннигиляцией запитать энергоустановку и вот тебе бесплатный источник энергии.
Погугли сколько стоит в долларах грамм антиматерии.
Аноним 28/09/22 Срд 09:39:38 #384 №565675 
>>565673
> Если это так просто, то почему ты этого еще не сделал?
Потому что я НЕ считаю, что это просто, я лишь не понимаю, почему это НЕ просто, почему примитивная логика, которую я привёл, не работает. Я реально не знаю, что тебе непонятно, сколько ещё раз нужно разжевать этот вопрос на разные лады. Ну не знаю, возьми химический конструктор и сооруди эту молекулу. Сложно было? Нет? А вот почему в химии это сложно?
> Это многое объясняет.
Поделись объяснением? Я уже это третий раз прошу. Только без перехода на личности, пожалуйста, (с этим велком в порашу /psy), а по теме вопроса.
Аноним 28/09/22 Срд 09:41:51 #385 №565676 
>>565674
> Погугли сколько стоит в долларах грамм антиматерии.
Потому что во вселенной её две песчинки (ой, уже столкнулись с чем-то). А тестостерон состоит из элементов, входящих в пятёрку самых распространённых. И на вид простой. Я же специально не сложный белок в пример привёл. Ну же.
Аноним 28/09/22 Срд 09:49:23 #386 №565677 
>>565675
>А вот почему в химии это сложно?
Потому что в химии это работает не так, как в конструкторе?
Если бы примитивная механика работала в химии, проблем бы вообще с синтезом не было.
Аноним 28/09/22 Срд 09:50:49 #387 №565678 
>>565676
>Потому что во вселенной её две песчинки
Нет. Потому что энергию, которую тебе придется затратить на ей производство, надо вырабатывать целой страной, типо США, пару сотен лет.
Аноним 28/09/22 Срд 10:18:04 #388 №565680 
>>565677
> Потому что в химии это работает не так, как в конструкторе?
Так, и почему именно не работает? По каким-то базовым принципам можно конкретизировать? Термодинамика, там всякое вот этот, хз. Как бы и сама вся химия это приближение к природе, глубокая аналогия. Любые понятия и представления рано или поздно расходятся с природой; конструктор - рано. Но всё же хоть что-то объясняет, в чём-то повторяет логику научных законов, природы (иначе нафиг он вообще нужен). Где пролегает граница этой аналогии? Ведь конструктор можно применять для объяснения не только статичной структуры, но и самой простой динамики взаимодействий? А где начинаются непреодолимые сложности, и синтез какой-то простой на вид молекулы ты уже никак не изобразишь конструктором (ну или с помощью лишь гораздо более сложных/многочисленных конструкций)? Т.е. в самых базовых случаях (водород+кислород=вода, например), ты можешь тупо присоединять по детальке (или уже здесь нет?) и это будет выглядеть правдоподобно, - а на каком этапе сложения этой молекулы тебе скажут: нет, так не происходит. И как ни пытайся по-другому сложить, нихуя не выйдет, потому что вступают в силу более абстрактные законы, на палочках-магнитиках не объяснишь. Хотя казалось бы, совсем немного же сложить, несколько этих шестиугольников и пару отростков, - но нет, упрёшься намертво в тупик.
>>565678
Но эта произведённая антиматерия тоже в пределах нашей вселенной находится, так что моё утверждение верно. Или если б её производить было дешевле, то её б не пару песчинок было, а хотя б с напёрсток.
Аноним 28/09/22 Срд 10:32:42 #389 №565681 
>>565680
> Или если б её производить было дешевле, то её б не пару песчинок было, а хотя б с напёрсток.
Сколько её конкретно в нашей вселенной никто не знает.
Но её проблема в том, что она аннигилирует при столкновении с простой материей, выделяя кучу энергии.
>>565680
>Ведь конструктор можно применять для объяснения не только статичной структуры, но и самой простой динамики взаимодействий?
После определенной очень низкой сложности уже нет.
Гугли задачу трех тел в общем виде.
Синтез любой молекулы ты можешь изобразить конструктором или на бумаге. От этого синтез не пройдет.
>>565680
>ты можешь тупо присоединять по детальке
Обычно синтез проводится в несколько стадий.
Чем больше стадий в синтезе, тем меньше конечный выход нужного продукта. Если ты тупо будешь присоединять, то от твоих исходных продуктов останется хуй да маленько на выходе.
Именно этим и занимаются химики-синтетики-экспериментаторы - они ищут ОПТИМАЛЬНЫЕ пути реакции и ОПТИМАЛЬНЫЕ условия для их протекания. На это положено немало жизней.
Аноним 28/09/22 Срд 11:19:12 #390 №565682 
>>564437 (OP)
Сап сосач! Я верю в то что местный анон еще не совсем отупел как я. Помогите решить задачку.
Есть цифра, к примеру это 500. Помогите найти формулу по которой из числа икс будет вычитаться определенный процент и в итоге должно получиться 500. То есть Х-6%=500. Как найти Х?
Заранее благодарю тебя анон.
Аноним 28/09/22 Срд 11:38:32 #391 №565683 
>>565682
Блядь. Это апефеоз тупости.
x·(1 - 0,06) = 500;
x= 500/0,94 ≈ 532.
Аноним 30/09/22 Птн 09:37:14 #392 №565708 
Подскажите как вкатиться в задачи по физике с переменной силой? Где про них посмотреть теорию, где задачи?
Аноним 30/09/22 Птн 10:47:11 #393 №565714 
>>565708
>задачи по физике с переменной силой
https://www.youtube.com/watch?v=kRH0v9_wT-M
Аноним 30/09/22 Птн 14:28:09 #394 №565716 
>>565714
Этого мало. Хочется какую-нибудь главу в учебнике или серию лекций, задачи.
Аноним 30/09/22 Птн 17:06:50 #395 №565720 
Расскажите все что вы знаете про сеть пассивного режима работы мозга?

На мой взгляд это очень интересная сеть, читая информацию про которую можно узнать много важного и создать пару любопытных гипотез.
Аноним 30/09/22 Птн 19:06:37 #396 №565722 
>>565720
Бессмысленная хня - смотрите, если не грузить мозг он работает по другому. Из серии - трава зелёная, небо голубое.
Аноним 30/09/22 Птн 19:34:53 #397 №565723 
>>565716
Интегралы второго рода брать умеешь?
Аноним 30/09/22 Птн 20:46:06 #398 №565725 
>>565723
Научусь по ходу
Аноним 01/10/22 Суб 00:19:36 #399 №565730 
Как много в мире известно аномальных мест/событий?

Какова вероятность существования в мире аномалий типа scp и служб их сокрытия?
Аноним 01/10/22 Суб 03:14:52 #400 №565737 
>>565730
50%
Аноним 01/10/22 Суб 04:07:20 #401 №565739 
>>565720
на стримах Алипова спроси или поищи у него
Аноним 01/10/22 Суб 12:46:37 #402 №565754 
ScLlSUl.png
Блять, почему нет нормальной единицы измерения магнитной силы типа подносишь пробную 1кг железку на 5 см к магниту и измеряешь силу притяжения? И сразу всё понятно. Нет, выдумаем лучше три единицы измерения, одна ебанутей другой
> в котором на 1 метр длины прямого проводника, перпендикулярного вектору магнитной индукции, с током силой 1 ампер действует сила 1 ньютон.
пиздец нахуй, кто вообще пробовал подносить магнит к проводам с током? а какой ток в железке течёт, и как ты его измеришь, умник? чисто выебнуться своими 180iq, мудила? в результате нихуя непонятно, 1 тесла это вообще много или мало, если поднести железяку к нему, оторвёт ли руки
Аноним 01/10/22 Суб 13:16:06 #403 №565758 
>>565754
Тесла это буквально то что ты хочешь, самая понятная и самая обычная.
>пиздец нахуй, кто вообще пробовал подносить магнит к проводам с током?
Никто этого не делает. Там формулировка такая, чтобы обозначить "Теслы можно пихать сразу во все физические формулы без пересчета, а еще, если что-то измеришь (тем же самым куском железа) то у тебя теслы вылезут автоматом).
Аноним 01/10/22 Суб 13:56:48 #404 №565762 
>>565758
окей, давай без приборов прикинь силу притяжения 1кг железки к магниту 1 тесла на расстоянии 10см. Я о том, что нет величины, отражающей самый практический случай. Согласись ведь, ну нихуя непонятно на взгляд, 1 тесла это много или мало; нихуя не скажешь, сколько тесл в кухонных магнитах, если я отрываю их от холодильника с силой 20грамм (ок, 0.2Н), допустим; что, для этого надо электрическую цепь сооружать, настраивать её на 1 ампер тока? А ещё непонятна динамика, 10 тесла ощущается в 10 раз сильнее, чем один, или зависимость нелинейная?
И ладно учёные, но даже никто из практиков, инженеров и просто любителей не придумал; зато придумали ещё 2 величины поизъёбистей. Железо есть дома у каждого, его параметры не нужно в справочниках искать.
Аноним 01/10/22 Суб 16:04:25 #405 №565765 
>>565762
>Согласись ведь, ну нихуя непонятно на взгляд
Очень понятно, но не для обывателя да. 1 Тесла это дохуя кстати.

И если тебе надо силу магнита измерять, то тебе нужен не Тесла, ни Вебер, это всё не для этого. Тебе нужна https://en.wikipedia.org/wiki/Maximum_energy_product
Аноним 01/10/22 Суб 16:22:48 #406 №565766 
>>565765
Так почему обыватели такую таблицу не составили для себя. Типа положить магнит данной намагниченности (типовой для магнитов данного материала, напр железного, неодимового, бытового электрического и тд), массы и формы на весы, под них железку и вычесть гравитацию из полученной силы. По-моему, для научпопа того же это весьма применимо. Короче, в итоге мне наверно нужно сведение к силе, весу, а не более абстрактные электромагнитные величины. Вот, например, новость читаю https://www.ixbt.com/news/2022/10/01/kitaj-stroit-samyj-moshnyj-v-mire-impulsnyj-magnit-ego-magnitnoe-pole-bolee-chem-v-dva-milliona-raz-silnee-chem-u-zemli.html откуда я знаю, какое оно у Земли, какой силе оно сопоставимо? У Земли-то оно слабое, а вот в миллионы раз больше это уже сильно или не очень? А если б по таблице мог сразу прикинуть, что это примерно как приложить 0.1см^3 неодимку, на столе валяющуюся, к железке, и полученную силу распределить в объёме 1л - и вот с такой примерно силой тянет Земля,- наверно многим бы понятней было.
Аноним 01/10/22 Суб 19:12:44 #407 №565771 
Какие есть практические задачи, с которыми и квантпуктер не справится? В которых для обработки в 100 раз большего числа данных потребуется в 2100 раз больше кубитов, например? Кроме очевидного "создать модель квантпуктера больше, мощнее и не сильно дороже текущего".
Аноним 01/10/22 Суб 19:14:04 #408 №565772 
в 2^100 раз больше кубитов
>>565771
Аноним 01/10/22 Суб 20:38:49 #409 №565777 
>>565766
>Так почему обыватели такую таблицу не составили для себя.
Потому что это им нахуй не упало? Не? Они поэтому и обыватели, их такое не интересует вообще
Аноним 01/10/22 Суб 21:12:52 #410 №565779 
>>565762
Ты вообще не понимаешь что такое магнитное поле
Это не давление в трубе, где взял площадь, умножил на давление и получил силу.
С какой стороны ты собрался магнит измерять? Под каким углом? Какая картина распределения магнитного поля у этого магнита? Ты вкурсе что разные материалы магнитятся сильно/слабо? Ты вкурсе что разной толщины и разного размера железки будут магниится с разной силой?
Если делать универсальные данные, то получишь такое же ебанутое условие с допущениями от которых ты выше горишь
Будет что-то типа "бесконечно большой, плоский кусок 100% чистого атомарного железа, с монокристаллической стрикторой, напротив магнита однородного магнита бесконечного размера(для ровности поля) создающего напряжённость магнитного поля х на 1 квадратный метр поверхности"
Аноним 01/10/22 Суб 21:37:08 #411 №565782 
16646191196680.jpg
Аноны, наш мозг работает с помощью электроимпульсов, верно? Тогда почему ЭМИ и другие электромагнитные поля не влияют на работу мозга?
Аноним 01/10/22 Суб 21:46:25 #412 №565784 
>>565782
>Тогда почему ЭМИ и другие электромагнитные поля не влияют на работу мозга?
Нипочему. Они влияют.
Аноним 01/10/22 Суб 21:50:25 #413 №565785 
>>565784
А где про это почитать можно? Я это все к чему просто, разве нельзя с помощью нейрошлема через направленное электромагнитное излученте воздействовать на определенные зоны мозга?
Аноним 01/10/22 Суб 21:59:27 #414 №565786 
>>565785
>разве нельзя
Можно. Что конкретно ты пытаешься сделать?
Аноним 01/10/22 Суб 22:02:10 #415 №565787 
>>565786
>Что конкретно ты пытаешься сделать?
Пока ничего, просто увидел в новостях нейролинк от Маска и задумался об альтернативных способах воздействия на мозг, нежели электроды в мозгу.
Аноним 01/10/22 Суб 22:26:57 #416 №565788 
>>565782
Вот кстати да, ионы же заряженные, наш мозг по пизде должен идти в магнитных полях сильных.
Аноним 01/10/22 Суб 23:01:02 #417 №565789 
>>565785
Нельзя, в мозгу нет металла, чтобы электромагнитное излучение индуцировать в потенциал.
Аноним 01/10/22 Суб 23:19:22 #418 №565791 
>>565789
У нас мозг на ионах работает, они заряженные.

Но это не имеет смысла, нужно управлять направлением, а это значит надо ставить охуеть мощный магнит буквально возле каждого синопса.
Они ж в случайном порядке распологаются, несколько магнитов, даже сотен, даже тысячи и стони тысяч никак не помогут управлять сознанием.
Аноним 01/10/22 Суб 23:32:55 #419 №565792 
>>565787
Шлем, управляющий указателем мыши, уже существует. Никаких непреодолимых препятствий нет.
Аноним 01/10/22 Суб 23:35:07 #420 №565793 
>>565788
>по пизде должен идти
Глюки и странные наблюдаются.
Аноним 01/10/22 Суб 23:36:48 #421 №565794 
>>565791
>Они ж в случайном порядке распологаются, несколько магнитов, даже сотен, даже тысячи и стони тысяч никак не помогут управлять сознанием.
Не в случайном. Они располагаются так, как удобно мозгу. Если дать мозгу полезную игрушку, которая всегда работает, он адаптируется сам.
Аноним 01/10/22 Суб 23:42:51 #422 №565795 
>>565794
Бред, ты миллиарды направлений хочешь перекрыть 1000 магнитов=урежишь мозг в 1000 раз, будешь как червь земляной, в сотни раз хуже самого больного клинического дауна.
Аноним 01/10/22 Суб 23:48:41 #423 №565796 
>>565792
Еблан, воздействовать и стимулировать зоны мозга, это не тоже самое, что наблюдать за активностями и по ним двигать курсор.
Аноним 02/10/22 Вск 07:46:04 #424 №565802 
>>565796
Наблюдение за курсором стимулирует мозг.
Аноним 02/10/22 Вск 09:09:43 #425 №565803 
000.jpg
Как подсчитать количество денег или ресурсов для выращивания ребенка?
Вообще нужно сравнить количество ресурсов для белого ребенка или черного из африки
Аноним 02/10/22 Вск 11:37:06 #426 №565807 
>>565803
ВВП на душу населения хорошо коррелирует с этим параметром.
Аноним 02/10/22 Вск 11:45:50 #427 №565808 
16155989433620.jpg
>>565807
Надо конкретно вырастить ребенка с нуля до 18 лет. ВВП слишком грубо
Аноним 02/10/22 Вск 12:05:35 #428 №565809 
>>565808
>Надо конкретно вырастить ребенка с нуля до 18 лет. ВВП слишком грубо
Кому надо, тот пусть и считает.
В конкретной стране затраты на конкретного ребенка будут отличаться на порядки - значительно больше, чем средние затраты от страны к стране. Поэтому грубая оценка здесь вполне оправдана.
Берешь ВВП по паритету покупательной способности, и окажется что вырастить ребенка в Люксембурге в сто раз дороже, чем в Нигере. Мне оценка кажется вполне здравой.
Аноним 02/10/22 Вск 12:49:38 #429 №565811 
>>565808
Так блять берёшь все цены и считаешь.
Человек может прожить на 250 кг зерна в год, это меньше 10$ в месяц.
Ребёнок растёт 20 лет, 240 месяцев=2400$, вот тебе и вся цена выращивания ребёнка в европе.
Аноним 02/10/22 Вск 13:01:39 #430 №565812 
>>565811
не, это полная хуйня уровня поделить ВВП
Физика фотона Вик 02/10/22 Вск 13:47:46 #431 №565813 
Если фотон передвигается со скоростью света, то следовательно для него не идёт время, т.е. время остановилось. Исходя из этого фотон существует вне времени или одновременно по всему времени. Как это так?
Аноним 02/10/22 Вск 14:21:48 #432 №565814 
Снимок.JPG
>>564437 (OP)
Что от меня требуется в задании?
Аноним 02/10/22 Вск 16:02:42 #433 №565815 
>>565814
уже сам делаю
Аноним 02/10/22 Вск 20:56:41 #434 №565825 
>>565813
Это значит что с его точки зрения он врезается в препятствие сразу же, как появляется. Хотя для нас может быть видно будто он летит миллиарды лет.

Но тут непонятно как он для нас изменяться, иметь вообще какое-то свойство типа частоты, полиризацию менять или что ещё у него за изменчивые свойства есть, если для него они не меняются.
И вообще понятие "сразу же", "мгновенно" очень сложные и неоднозначные, а для фотона именно что строго "мгновенно" должно это происходить. А такого не бывает.
Аноним 02/10/22 Вск 20:58:22 #435 №565826 
>>565812
Ты тупой, все цены это и есть все цены, это самая что ни на есть та самая "сооимость" о которой ты говоришь. Стоимость складывается из всех трат.
Аноним 02/10/22 Вск 22:38:10 #436 №565832 
То, что стоимость воспитания ребенка прямо пропорциональна душевому ВВП (ППС) - это т. н. "нулевая гипотеза", т. е. самое простое потенциально рабочее предположение. Соответственно, если эта гипотеза не устраивает, нужно объяснять почему. Нужно сказать, что эта стоимость отличается а Люксембурге и Нигере не в 100 раз, а в 5 или в 500, и доказать это.
Не хочешь опровергать и доказывать - иди на хуй.
Аноним 02/10/22 Вск 23:08:24 #437 №565834 
подскажите обычные силиконовые перчатки из дефолтного хоз магаза защитят от концентрированной H2SO4? (92 % техническая)
Если нет то какие нужно брать чтобы наверняка?
Аноним 02/10/22 Вск 23:10:19 #438 №565835 
>>565834
>силиконовые
точнее латексные
Аноним 03/10/22 Пнд 02:35:34 #439 №565842 
>>565834
>>565835
Латекс вполне держит. Даже обычная резина держит. Но тут зависит от качества. Кислота отнимает воду, и если в резине влажные микропоры, то она медленно в труху превращается.
С силиконами сложнее, есть кислотостойкие, есть нет.
Аноним 03/10/22 Пнд 13:43:45 #440 №565853 
16637444321861.jpg
>>564437 (OP)
Чем можно легально упороться? Никогда не курил, никогда не пил, только несколько раз в общаге кальян с травой курили.

Что можете легального посоветовать такого? Хочется иногда вечером забыться, алкоголь не предлагать.
Аноним 03/10/22 Пнд 13:46:44 #441 №565855 
>>565853
Мускатный орех
Аноним 03/10/22 Пнд 13:49:02 #442 №565856 
>>565855
Кайф от мускатного ореха под вопросом и длится слишком долго эффект.
Аноним 03/10/22 Пнд 16:08:52 #443 №565869 
>>565856
В очко себя дилдаком подолби, пидор.
Гляди-ка, нужно и дешево, и легально, и кайф надёжный, и контролируемо, и безвредно.
Если бы такое было, человечество давно бы упоролось и сдохло.
Аноним 04/10/22 Втр 15:28:45 #444 №565904 
w-rings.webm
вопрос про зрение
человек видит в зеркале и в мониторе то же, что ирл
а кошки/собаки/попугаи/рыбки?
Аноним 04/10/22 Втр 16:00:09 #445 №565905 
>>565904
"Ирл" это только перпендикулярная проекция парралельными лучами.
Тебе кажется что ты видишь то что есть в реале, потому что когда ты приближаешь руку к чему-то, чтобы потрогать рельно ли предмет и именно там где ты видишь и такой ли он формы, то при сближении предмета и руки иллюзии взаимно деформируются до полного совпадения при касании.
Аноним 04/10/22 Втр 16:05:37 #446 №565906 
>>565905
поправляюсь
>человек видит в зеркале и в мониторе то же, что видит ирл
вопрос про зрение, не про соответствие реальности восприятию
Аноним 04/10/22 Втр 17:04:06 #447 №565909 
>>565906
Непонятно что ты уточнил, надо перефразировать. Ну зеркало, ну монитор, и что?
Аноним 04/10/22 Втр 18:31:32 #448 №565915 
Пару лет назад подтвердили существование гравитационных волн. Подтверждает ли это что гравитация это именно сила(незримое воздействие) а не кривизна пространства или наоборот ?
Аноним 04/10/22 Втр 20:23:18 #449 №565918 
>>565814
Ну тип, есть или нет
Аноним 04/10/22 Втр 20:29:28 #450 №565920 
>>565664
Проблемы хим синтеза в низком выходе целевого продукта и дохуя всякого говна, можно выделить из той же мембраны клеток, например эритоцитов из крови какого-нибудь животного и на вжэх или гжх прогнать.

Выйден дешевле, но опять же, не в районе 100 рублей
Аноним 05/10/22 Срд 00:11:23 #451 №565929 
>>565915
Подтвердили именно ОТОшную версию - гравитация не сила/поле, а искривление пространства-времени.
Аноним 06/10/22 Чтв 16:00:13 #452 №566019 
Пришел к вам с идеей карликовых животных, взрослые особи которых слишком массивные.

Возможно ли остановить рост животного на уровне 2-5 месяцев? Будет ли от этого вред самому животному? Как при этом сохранить продолжительность его жизни?

Представьте карликовых слонов, жирафов, бегемотов, крокодилов. Которые помещаются прямо дома
Аноним 06/10/22 Чтв 16:02:49 #453 №566020 
>>565853
Зверобой, кофе, шоколад, ромашка, квас из цикория, чайный гриб, морской рис
Аноним 06/10/22 Чтв 16:09:17 #454 №566021 
>>566019
Лучше селекцией делать. Вон каких собачек сделали разных.

Но тут проблема устойчивости генетического кода. Кошек например намного сложнее селекцией изменять, у них слишком стабильный генетический код.
Аноним 07/10/22 Птн 10:04:32 #455 №566044 
1665126270039.jpg
1665126270040.jpg
1665126270041.jpg
Какой формулой описывается ВАХ фотодиода при внешнем фотоэффекте? В методичке к лабам нам был дан только график пик1. Я сходу подумал про арктангенс, у него даже асимптота горизонтальная есть для тока насыщения. А для замыкающего напряжения его можно чуть поднять, на втором пике построил график I=atan(x)+1. Если его чуть растянуть, подходит идеально.
Только вот проблема есть в физическом смысле такой зависимости, откуда здесь вообще может взяться арктангенс? У меня есть гипотеза, что эта зависимость выходит из какого-нибудь диффура, потому что арктангенс это интеграл более менее нормальной величины (1/(x²+1)) Но дальше раскручивать мне не хватает знаний. Ничего даже близкого к диффурам нам препод на лекции по фотоэффекту не рассказывал.
Препода который лабы ведет спросил по этому поводу, он ответил что все херня и случайность, но меня такой ответ не устраивает. Измерения с лабы (пик3) превосходно описываются формулой I=k×atan(c×U+d)+b при некоторых константах k, b, c, d (Примерно 13.68atan(0.36U-0.38)+7). Не совсем точно предсказывает ток насыщения, но это потому что для приближения к асимптоте арктангенса с малой погрешностью понадобятся огромные напряжения. Зато замыкающее напряжение предсказано очень точно.
Аноним 07/10/22 Птн 10:37:57 #456 №566046 
>>566044
>Какой формулой описывается ВАХ фотодиода
Наверняка это никакой не арктангенс, а логистическая кривая.
Аноним 07/10/22 Птн 10:56:19 #457 №566047 
image.png
>>566044
Очень похоже на уравнения для ВАХ для электролиза, там диффузию когда учитываешь, получается такая шляпа в общем виде. Физический смысл констант тебе наверное особо не нужен, поэтому я их просто a b c взял, там всякие предельные токи диффузии и прочая хуетень
Аноним 07/10/22 Птн 11:02:44 #458 №566048 
>>566047
Это и есть логистическая кривая. Она вылазит при решении всяких диффуров часто.
Аноним 07/10/22 Птн 11:18:46 #459 №566049 
>>566046
>>566047
Очень плохо работает для задерживающих напряжений (U<0), замыкающее предсказывает раза в два меньше экспериментального.
Аноним 07/10/22 Птн 13:37:30 #460 №566058 
>>564437 (OP)
Пусть есть множество S = { A, notA}, где A - множество каких-то элементов, а notA - множество множеств, которые не являются A,
вопрос, какие элементы тогда содержатся в множестве notS?
notS = {?????}

поясните за теорию множеств
Аноним 07/10/22 Птн 14:03:21 #461 №566059 
>>565826
Тупорылый еблан, мир состоит не из денег, а из материалов, то есть ресурсов. Об этом и речь
Аноним 07/10/22 Птн 14:04:17 #462 №566060 
>>565832
на моем хую твоя мамаша скачет
Аноним 07/10/22 Птн 14:06:10 #463 №566061 
>>566058
notS = ø
Аноним 07/10/22 Птн 14:08:31 #464 №566062 
>>566061
ø ∈ notA
Аноним 07/10/22 Птн 14:10:17 #465 №566063 
>>566062
Тогда notS не существует
Аноним 07/10/22 Птн 14:12:10 #466 №566064 
>>566058
Подумай еще, входит ли S в notA, охуеешь
Аноним 07/10/22 Птн 14:13:22 #467 №566065 
>>566061
>>566063
>>566064
вообще ты прав notS = ø
Я неправильно вопрос сформулировал и запутался
Аноним 07/10/22 Птн 14:15:45 #468 №566066 
>>566064
теория множест вообще жива или на нее хер забили?
Где че почитать?
Аноним 07/10/22 Птн 14:21:13 #469 №566067 
>>566066
Наверняка жива, как и классическая физика. Работает не везде, но где удобно - применяется. Литературы подсказать не могу.
Аноним 07/10/22 Птн 15:45:08 #470 №566071 
>>566059
Тупорылый долбаёб, так ты можешь посмотреть что и чего тратится ПО ЦЕНАМ.
Аноним 07/10/22 Птн 19:53:54 #471 №566086 
Откуда клетка а потом ембрион, маленький ребенок, знает в какую сторону и сколько расти?
Аноним 07/10/22 Птн 20:14:15 #472 №566088 
>>564437 (OP)
Как найти данные по численности и распределению определенного вида?
Аноним 07/10/22 Птн 20:58:22 #473 №566089 
>>566086
Сначала вес и давление, потом всякие хим и электро сигналы говорят разным областям расти/не расти.
Аноним 07/10/22 Птн 21:12:27 #474 №566091 
>>566086
Как зерно прорастает, рецепторы определяют клетке кем ей стать - корнем или стебельком.
У животных так же, только вариаций клеток и сигналов на порядки больше.
Аноним 07/10/22 Птн 21:26:19 #475 №566092 
>>566091
Наверное у него вопрос про более сложное и точное определение формы.
Типа органы из клеток одной ткани состоят практически (допустим), но форма сложная, и у всех людей прям очень одинаковая форма органов.
Аноним 07/10/22 Птн 22:00:25 #476 №566094 
>>566092
А у растения по твоему простые формы - листки, цветы, пестики-тычинки?
Я дал направление (начать с простого) и дал описание простейшего инструмента. ДНК с кодировкой всего организма находится в каждой клетке, а конкретная клетка исполняет часть кода, которая для неё становится основной и конечной.
В коде ДНК заложены формы органов и тела, а клетки взаимодействуют друг с другом, тем и устанавливается форма, т.е. растут и делятся до определённого ДНК предела.
Аноним 07/10/22 Птн 23:31:52 #477 №566095 
>>566094
Ты так говоришь будто клетки одной и той же ткани разные.
Но они одинаковые.
Тут нужен какой-то обмен информацией обоюдный, но клетки этого не умеют.
Аноним 08/10/22 Суб 01:00:07 #478 №566096 
>>566095
>Тут нужен какой-то обмен информацией обоюдный, но клетки этого не умеют
Умеют, конечно же. Идёт постоянный обмен сигнальными молекулами. От градиента их концентрации зависит, как именно будет прочитан генетический код.
Аноним 08/10/22 Суб 06:18:12 #479 №566099 
16649783070880.webm
Психологи есть? Объясните реакцию лоли
Аноним 08/10/22 Суб 08:06:43 #480 №566101 
>>566099
Адреналин.
Аноним 08/10/22 Суб 08:32:20 #481 №566103 
image.png
>>566101
Точно, древние инстинкты щелкнули
Аноним 09/10/22 Вск 15:11:43 #482 №566125 
Какие основные факторы мешают животным выращивать большие мозги?
Почему большой мозг животных не особо влияет на развитие
Аноним 09/10/22 Вск 15:17:07 #483 №566127 
>>566125
> Какие основные факторы мешают животным выращивать большие мозги?
Отсутствие прямохождения.

>Почему большой мозг животных не особо влияет на развитие
Потому что важно отношение объёма мозга к массе тушки.
Аноним 09/10/22 Вск 15:26:16 #484 №566129 
>>566127
>Потому что важно отношение объёма мозга к массе тушки.
Как это должно работать?
>Отсутствие прямохождения.
Слону не мешает. У мелких животных соотношение мозга к телу больше чем у человека
Аноним 09/10/22 Вск 15:46:37 #485 №566130 
>>566129
>Слону не мешает.
Зачем вообще слонам большие мозги? Они же ничего не делают. Они постоянно играют в 4d шахматы за возможность спариться и передают ревом команды?
Аноним 09/10/22 Вск 15:52:15 #486 №566131 
слонопотам.mp4
>>566130
>Они же ничего не делают
Целый день ходят. 40 км преодолевают
Тут видно что он пытается соорудить себе рог носорога, а потом кидает палку во врага как человек кидает рукой. Вангую через пару миллионов лет, он мог топоры и копья строгать прямо у себя на бивнях
Аноним 09/10/22 Вск 17:09:45 #487 №566138 
>>566131
>Вангую через пару миллионов лет,
Нет задач.
А резкая смена климата и условий, которая могла бы послужить этой самой "задачей" для выживания просто убила бы их, т.к. ноги их не способны нормально реализовать развитый мозг для выживания.
Хобот... одного мало, даже если там на конце хобота идеальная рука вырастет с подобиями пальцев.
То что вырастет второй хобот-маловероятно, слишком сильная мутация, долго времени, уже вымрут при изменении климата(чтобы была мотивация вообще к каким-то изменениям).

Они в тупике эволюционном.
Вот всякие лемуры, еноты, сурки.. У них лапы очень человеческие на вид, сильно на руку похожи, и они ими много и активно пользуются, плюс часто на задние лапы встают и ходят. И поведение очень человеческое.
С одной стороны как бы уже есть обезьяны... Казалось бы. Но все обезьяны что есть лазают по деревьям в основном, и живут в лесах, а значит из рука не улучшится для использования мозга.(хвататься надо за ветки)

А вот сурки, еноты, лемуры, они степные, или по крайней мере на земле живут, более человекоподобные условия жизни. Я бы сказал что у них шанс стать следующими активно использующими инструменты и увеличивающими свой мозг даже выше чем у обезьян.
Аноним 09/10/22 Вск 17:14:10 #488 №566139 
>>566138
>Но все обезьяны что есть лазают по деревьям в основном, и живут в лесах
Собакомордые бегают по саване толпами. Но пока человек живет на земле, он им не даст подняться выше
Аноним 09/10/22 Вск 17:33:53 #489 №566140 
>>566139
>Но пока человек живет на земле, он им не даст подняться выше
Довольно много есть ещё пока куда расти в эволюционном плане. Человек много где не лезет, пока они стаями не пытаются атаковать города с копьями и пока не добывают ресурсы.
Человек ресурсы добывать начал тоже только 5к лет назад нормально, так чтобы это заметно было.
Они прям очень сильно похожими в способностях на человека могут стать, пока упрётся всё проблему конкуренции с человеком.

У них же вроде как-то застопорилось всё уже давно? Давно не видно подвижек эволюционных.
Аноним 09/10/22 Вск 17:43:09 #490 №566142 
>>566140
> У них же вроде как-то застопорилось всё уже давно? Давно не видно подвижек эволюционных.

Для подвижек нужны условия отбора, а им и так хорошо. Если они существуют и не вымирают, значит эволюционно успешны.
Аноним 09/10/22 Вск 18:00:16 #491 №566143 
>>566142
Это понятно, но у людей переход с жизни в лесах к жизни в полях послужил стимулом к изменениям которые не остановились до превращения в человека.
А у этих чё остановилось как думаешь?

Хотя я не уверен насколько уже они долго без изменений, может совсем не долго после превращения из лесных в таких, и на самом деле изменения идут дальше.

Хотя они скорее превращаются в четвероногих вместо прямозодящих, и больше похожих на лошадей/собак.
Аноним 09/10/22 Вск 18:03:18 #492 №566145 
>>566140
Человек перебил всех, кто конкурировал с ним. Рядом не осталось никого.
>Довольно много есть ещё пока куда расти в эволюционном плане.
Если бы не угроза вымирания
Аноним 09/10/22 Вск 18:14:09 #493 №566146 
>>566143
> но у людей переход с жизни в лесах к жизни в полях послужил стимулом к изменениям которые не остановились до превращения в человека.

У эволюции нет стимула. Эволюция это вымирание не приспособившихся к изменившимся условиям окружающей среды вариантов мутаций. От обезьяны до человека пройдено бесчисленное количество бутылочных горлышек - условий отбора. После которых эволюция останавливается до следующего бутылочного горлышка.
Аноним 09/10/22 Вск 18:16:37 #494 №566147 
>>566146
>У эволюции нет стимула. Эволюция это вымирание не приспособившихся
Есть постоянно случайные мутации, в том числе более умных, с бОльшим мозгом и более приспособленными конечностями для инструментов.
Непонятно почему они не закрепляются у них так активно как это было у человека. Условия те же.
Аноним 09/10/22 Вск 18:47:45 #495 №566150 
>>566147
Выше писал, всех прогрессивных отпиздил и вытеснил гомосапиенс. С близкими частично ассимилировался, но в основном истреблял. То что сейчас наблюдаем, это занятые ниши, на которые человек не претендует. На планете нет место для двух разумных видов, и проявления расизма наглядно это демонстрируют. Если мы истребляли хищников палками и копьями, то себе подобных тем более, так как человек самый опасный хищник.
Аноним 09/10/22 Вск 18:50:21 #496 №566151 
>>566147
> Есть постоянно случайные мутации, в том числе более умных, с бОльшим мозгом и более приспособленными конечностями для инструментов.
Во первых без жесткого отбора это очень медленный процесс, надо что бы мутация закрепилась и распространилась на весь вид. Во вторых не факт что новая мутация пройдёт половой отбор, для самок урод мутант=омега. Поэтому без горлышка отбора эволюция идёт очень медленно, природа работает по принципам - не мешай механизму работать; новое это риск, а старое уже проверено. Крокодилы, акулы и всякие муравьи гарантируют.

А вот горлышко отбора даёт мощный толчок эволюции, если в связи с изменением окружающей среда мутант выживает, а остальные вымирают, то его гены мгновенно распространяются и закрепляются.

Это как селекция, если собачек отбирать по какому нибудь признаку, а остальных уничтожать, то вид меняется очень быстро - естественным путём такого не добиться.
Аноним 09/10/22 Вск 22:39:28 #497 №566155 
>>566150
>На планете нет место для двух разумных видов, и проявления расизма наглядно это демонстрируют.
В будущем межнациональных конфликтов не будет.
мимо китаец
Аноним 09/10/22 Вск 22:42:49 #498 №566156 
>>566155
У вас там территориальных конфликтов выше крыши. Вам бы современное оружие изобрести, а как былые времена придут япошки и мелко порежут
Аноним 10/10/22 Пнд 07:06:19 #499 №566164 
перекат.gif
https://2ch.hk/sci/res/566163.html
Аноним 10/10/22 Пнд 18:49:39 #500 №566177 
>>566156
Весьма вероятно
Аноним 11/10/22 Втр 11:35:40 #501 №566199 
изображение.png
>>564437 (OP)
Могли бы они существовать?
С перестроенной нервной системой, без пищевода, ради усиления мышечной массы у лучевых костей.
Такие быстрые метаморфозы с помощью патогена реально невозможные, как и изменение костной структуры каким-то патогеном, хоть и есть болезни, которые изменяют костную массу.
Аноним 11/10/22 Втр 11:35:51 #502 №566200 
>>566199
Да
Аноним 12/10/22 Срд 10:24:09 #503 №566223 
16577557864942.png
>>565369
Кстати, этот пост навеял странный вопрос и даже немного извращенный:
могло бы появиться потомство у нашего древнего предка типа австралопитека и у современного человека?

Вроде гены должны совпадать по большей части, не?
Аноним 12/10/22 Срд 15:59:59 #504 №566233 
>>566223
Я не уверен но скорее всего существуют челы с 60% сходством генов.
Они не пропали бесследно
Аноним 16/10/22 Вск 10:57:22 #505 №566367 
>>566223
Этого недостаточно. Лучше иметь потомство с кроманьонцем. Учёные говорят фенотип сформировался давно, а мозг тренировался очень очень долго. Представляете человека с повадками обезьяны которому 10 ки тысяч лет нужно чтобы начать одеваться, разговаривать и т д
Аноним 16/10/22 Вск 11:03:18 #506 №566368 
>>565834
Проверь перчатки перед использованием. Капни кислотой. Некоторые перчи разьедаются если не убирать капли.
Аноним 02/11/22 Срд 08:55:18 #507 №567188 
>>566223
Можно название фотосета с этоё фоткой?
comments powered by Disqus

Отзывы и предложения